0% found this document useful (0 votes)
189 views93 pages

Block 2 Sample - Medico Express

The 'Medico Express: Musculoskeletal & Locomotion' is a comprehensive textbook designed for 1st Year MBBS students, covering essential subjects in human anatomy, physiology, and pathology. It includes features like past MCQs, essay questions, and quick summaries to aid in exam preparation and practical understanding. The first edition for the academic year 2024-2025 reflects the latest educational methodologies and aims to enhance the learning experience of medical students.

Uploaded by

sherazahmadlive
Copyright
© © All Rights Reserved
We take content rights seriously. If you suspect this is your content, claim it here.
Available Formats
Download as PDF, TXT or read online on Scribd
0% found this document useful (0 votes)
189 views93 pages

Block 2 Sample - Medico Express

The 'Medico Express: Musculoskeletal & Locomotion' is a comprehensive textbook designed for 1st Year MBBS students, covering essential subjects in human anatomy, physiology, and pathology. It includes features like past MCQs, essay questions, and quick summaries to aid in exam preparation and practical understanding. The first edition for the academic year 2024-2025 reflects the latest educational methodologies and aims to enhance the learning experience of medical students.

Uploaded by

sherazahmadlive
Copyright
© © All Rights Reserved
We take content rights seriously. If you suspect this is your content, claim it here.
Available Formats
Download as PDF, TXT or read online on Scribd

&

© All rights reserved to the Author & Publisher. This book is protected by
copyright Act. No part of this publication can be reproduced, stored or
retrieval system, or transmitted in any form or by any means, electronic,
mechanical, photocopying, recording or otherwise, without prior
permission of the copyright owners.

Pioneers Law Firm

Advocate High Court, Lahore - Pakistan

MEDICO EXPRESS REVIEW BOCK 2

Dr. Hunzala Siddiq Khan

Dr. Fatima Nasir

Medico Express Publishers


It gives us immense pleasure to present the "Medico Express: Musculoskeletal &
Locomotion," a comprehensive and meticulously crafted resource tailored for 1st Year
MBBS students. This detailed review textbook, part of the Medico Express series, is
designed to meet the evolving needs of medical students as they embark on their journey
through the intricate world of human anatomy, physiology, and pathology.
The medical field demands not only theoretical knowledge but also the ability to apply this
knowledge practically. Keeping this in mind, this textbook covers an extensive range of
subjects including Gross Anatomy, Embryology, Locomotion, Histology, Physiology,
Biochemistry, Pathology, Musculoskeletal, Pharmacotherapeutics, Aging, and Disease
Prevention and Impact. Each subject has been meticulously detailed to ensure that
students grasp the fundamental concepts thoroughly and are able to build a solid
foundation for their medical careers.
Key features of this textbook include:
PAST MCQs Pearls: A rich collection of multiple-choice questions to help students test their
understanding and prepare for exams effectively.
PAST UHS SEQs: A compilation of short essay questions from previous University of Health
Sciences (UHS) exams, providing insights into the type of questions that have been asked
historically and how to approach them.
Past Solved OSPE: Detailed solutions to past Objective Structured Practical Examination
(OSPE) questions, aiding students in understanding the practical aspects of the subjects
covered.
Solved Pearls: Step-by-step solutions and explanations for complex topics and questions,
enabling students to overcome challenging areas with ease.
Quick Summaries: Concise summaries of each chapter, offering a quick revision tool to help
students recall key points and concepts swiftly.
The Medico Express series is committed to enhancing the learning experience of medical
students by providing high-quality educational resources that are both informative and
engaging. This first edition for the academic year 2024-2025 reflects our dedication to
staying updated with the latest advancements and educational methodologies in the
medical field.
We would like to express our gratitude to the team of experts, educators, and medical
professionals whose tireless efforts have made this textbook possible. We also extend our
thanks to Zubair Book Depot for their support in bringing this book to life.
As you navigate through the pages of this textbook, we hope it serves as a valuable tool in
your academic journey and helps you achieve your goals. Your feedback and suggestions
are always welcome as they help us improve and better serve the educational community.
Wishing you all the best in your studies and future medical endeavors.
The Authors
Dr. Hunzala Siddiq Khan (University College of Medicine & Dentistry)
Dr. Fatima Nasir (Akhtar Saeed Medical and Dental college)
Dr. Eisha Ishaq (Avicenna Medical College)

Dr. Nouraiz (Abwa Medical College, Khurrianwala)


Dr. Amina Tahira (Khawaja Muhammad Safdar Medical college)
Dr. Mahrukh Ali (Akhtar Saeed Medical and Dental college)
Dr. Muhammad Abdul Muqtadir Qureshi (Quaid-e-Azam Medical College)
Dr. Sana Iftikhar(Sheikh Zayed Medical College and Hospital)
Dr. Sadia Javaid (Quaid-E-Azam Medical College)
Dr. Usman Fayyaz (Quaid-E-Azam Medical College)
Dr. Hafiz Shahbaz Zahoor (Quaid-E-Azam Medical College)
Dr. Maheen Hamid (Multan Medical and Dental College)
Dr. Nimra Faisal (Sialkot Medical College)
Dr. Haram Sajid (Akhtar Saeed Medical and Dental college)
TABLE OF CONTENTS

BLOCK 2 MODULE -03

MUSCULOSKELETAL & LOCOMOTION


Code Topic Pg no.
GROSS ANATOMY
UPPER LIMB
MS-A-001 Pectoral Region 1
MS-A-002 Dermatomes and cutaneous innervation of Upper Limb 4
MS-A-003 Pectoral region & Back 8
MS-A-004 Bones of Upper Limb: Clavicle & Scapula 11
MS-A-005 Bones of thorax, Joints of Upper Limb: Sternoclavicular Joint 17
MS-A-006 Axilla 18
MS-A-007 Bones of upper limb: Humerus 25
MS-A-008 Joints of Upper Limb: Shoulder Joint 30
MS-A-009 Rotator Cuff 35
MS-A-010 Nerves of Upper Limb 38
MS-A-011 Blood supply of arm 50
MS-A-012 Muscles of Arm 52
MS-A-013 Bones of Forearm 56
MS-A-014 Muscle of Anterior/Flexor Compartment of Forearm 61
MS-A-015 Muscle of Lateral and Posterior/Extensor Compartment of Forearm 65
MS-A-016 Nerves of Forearm 67
MS-A-017 Blood supply of forearm 71
MS-A-018 Retinacula of Forearm 76
MS-A-019 Carpal tunnel Syndrome 78
MS-A-020 Forearm: Blood supply and Venous drainage; Joints of Upper Limbs: Elbow Joint 80
MS-A-021 Joints of Upper Limbs: Elbow Joint 83
MS-A-022 Joints of Upper Limbs: Radioulnar Joint 84
MS-A-023 Interosseous membrane 87
MS-A-024 Fascia & Muscles of Hand 88
MS-A-025 Hand & Actions of Muscles of Upper Limb as a Functional Unit 91
MS-A-026 Blood vessels of forearm and hand 100
MS-A-027 Nerves of forearm and hand 104
MS-A-028 Joints of Hands 105
MS-A-029 Skills 107
PAST MCQS 115
LOWER LIMB
MS-A-030 Hip Bone 132
MS-A-031 Femur 141
MS-A-032 Fascia Lata 150
MS-A-033 Neurovascular Supply of Thigh 152
MS-A-034 Femoral Triangle & Canal 1527
MS-A-035 Muscles of Anterior Compartment of Thigh 161
MS-A-036 Neurovascular Supply of Anterior Compartment of Thigh 163
MS-A-037 Adductor Canal 166
MS-A-038 Muscles of Medial Compartment of Thigh 167
MS-A-039 Neurovascular Supply of Medial Compartment of Thigh 169
MS-A-040 Gluteal Region 173
MS-A-041 Muscles of Posterior Compartment of Thigh 179
MS-A-042 Blood Supply of Posterior Compartment of Thigh 181
MS-A-043 Sciatic Nerve 183
MS-A-044 Hip Joint 186
MS-A-045 Popliteal Fossa 190
MS-A-046 Knee Joint 194
MS-A-047 Muscles of Leg 207
MS-A-048 Neurovascular Supply of Leg 210
MS-A-049 Flexor, Extensor, and Peroneal Retinacula 214
MS-A-050 Tibio-fibular Joint 217
MS-A-051 Ankle Joint 219
MS-A-052 Plantar Fascia 222
MS-A-053 Muscles of Foot 224
MS-A-054 Small Joints of Foot 230
MS-A-055 Arches of Foot 231
MS-A-056 Retinacula of Foot 233
MS-A-057 Neurovascular Supply of Foot 235
MS-A-058 Arterial and Venous Drainage of Lower Limb 239
MS-A-059 Human Gait 241
MS-A-060 Lymphatic Drainage of Lower Limb 243
MS-A-061 Cutaneous Dermatomes & Nerve Supply of Lower Limb 244
MS-A-062 Topographical and Radiological Anatomy of Lower Limb 248
MS-A-063 Bone Fracture 253
MS-A-064 Joint Dislocation 262
PAST MCQS 268
EMBRYOLOGY
MS-A-065 Development of Muscles 283
MS-A-066 Development of Limb 285
MS-A-067 Development of Nerve Supply of Limbs 289
MS-A-068 Congenital Anomalies of Limbs 291
MS-A-069 Development of Cartilage 293
PAST MCQS 296
HISTOLOGY
MS-A-070 Histology of Muscles 299
MS-A-071 Functional Histology 307
MS-A-072 Histology of Osseous tissue 309
MS-A-073 Histology of Bone 314
MS-A-074 Functional Histology of Bone 317
MS-A-075 Histology of Cartilage 318
MS-A-076 Mechanism of Bone Growth 321
PAST MCQS 322
PHYSIOLOGY
MS-P-001 Diffusion / Equilibrium Potentials 325
MS-P-002 Nernst potential 326
MS-P-003 Goldman Equation 327
MS-P-004 Resting Membrane Potential in Neurons 329
MS-P-005 Neurons 331
MS-P-006 Classification of Neurons & Fibers 334
MS-P-007 Action Potential of Neurons 335
MS-P-008 Role of other ions in action potential 338
MS-P-009 Local / Graded potentials 339
MS-P-010 Synapse 341
MS-P-011 Conduction of Nerve Impulse 343
MS-P-012 Nerve Degeneration 344
MS-P-013 Skeletal muscle 346
MS-P-014 Characteristics of whole muscle contraction 350
MS-P-015 Mechanics of muscle contraction 352
MS-P-016 Neuromuscular junction 355
MS-P-017 Smooth Muscle 361
PAST MCQS 369
BIOCHEMISTRY
MS-B-001 Classification carbohydrates 383
MS-B-002 Carbohydrates 385
MS-B-003 Extracellular matrix 387
MS-B-004 Glycolysis and Tricarboxylic acid cycle (TCA) 395
MS-B-005 Protein Digestion & Transport across cell 398
MS-B-006 Reactions involve in catabolism 400
MS-B-007 Transportation of ammonia to liver 402
MS-B-008 Urea cycle 404
MS-B-009 Urea cycle 406
MS-B-010 Protein metabolism 407
MS-B-011 Inborn errors of amino acid metabolism 409
PAST MCQS 413
PATHOPHYSIOLOGY AND PHARMACOTHERAPEUTICS
MS-Ph-01 Drugs acting on Neuromuscular Junction (NMJ) 419
MS-Ph-02 Drugs in Myasthenia Gravis 431
MS-Ph-03 Local Anesthetics 432
MS-Pa-01 Muscle remodeling 434
MS-Pa-02 Diseases of Muscle 436
MS-Pa-03 Diseases of Bone 439
MS-Pa-04 Disease of Cartilage 442
PAST MCQS 444
DISEASE PREVENTION AND IMPACT
MS-CM-001 Back Pain 447
MS-CM-002 Work-related Musculoskeletal disorders (MSD) 449
MS-CM-003 MSD related to mobile usage 452
MS-CM-004 Ergonomics 455
MS-CM-005 Noncommunicable Musculoskeletal diseases 456
MS-CM-006 Risk factor assessment of Musculoskeletal diseases 458
MS-BhS-001 Psychosocial factors influencing chronic illnesses 459
MS-BhS-002 Psychosocial Impact of Disease and its management 460
PAST MCQS 462
AGING
MS-Ag-01 Bone (Effect of age on bone fragility) 464
MS-Ag-02 Cartilage (Effect of age on resilience) 465
MS-Ag-03 Muscle (Effect of age on muscular strength) 466
MS-Ag-04 Effect of estrogen on Bone Mineral Density 467
PAST MCQS 467
PAST OSPE 469
PERLS 497
Medico Express Block 2 GROSS ANATOMY

GROSS ANATOMY
UPPER LIMB
MS-A-001 Human Anatomy (Pectoral Region) Learning Objectives
Describe the topographical anatomy of the Pectoral Region
Dissect the Pectoral Region or use models to identify the key structures
Describe the muscles of the Pectoral Region with their origin, insertion, nerve supply, and actions.
PECTORAL REGION
• The pectoral region lies on the front of the chest.
• consists of structures that connect the upper limb to the anterolateral chest wall.
• The mammary gland lies in this region.
Superficial Fascia:
Contents of Superficial Fascia:
• In addition to fat.
Cutaneous Nerves of Pectoral Region:
• Cutaneous Nerves (from cervical plexus and intercostal nerves)
• Medial, Intermediate and Lateral Supraclavicular Nerves (Branches of cervical plexus C3, C4)
• Anterior and Lateral cutaneous Branches of Intercostal nerves (T2-T6)
Cutaneous Vessels of Pectoral Region:
• Perforating branches of the internal thoracic artery
• Cutaneous Branches (from internal thoracic and posterior intercostal arteries)
• Lateral cutaneous branches of posterior intercostal arteries.
The Platysma:
• Location: lies on the anterior aspect of the neck.
• Supply: Supplied by branch of facial nerve EXPRESS HIT
• Function: Wrinkles the skin of the neck and protects ts external jugular Intercostobrachial nerve (T2):
vein Although it's a cutaneous nerve, it
THE BREAST: often gets invaded or compressed
• Occurrence: The breast is found in both sexes, but is rudimentary in the male. early in breast cancer — which
• Development: It is well developed in the female after puberty. explains unexplained medial
• Nature: The breast is a modified sweat gland. upper arm pain or numbness in
• Importance: It forms an important accessory organ of the female reproductive pre-diagnostic stages
system, and provides nutrition to the newborn in the form of milk.

DEEP FASCIA:
• Superiorly: The deep fascia covering the pectoralis major muscle is called the pectoral fascia.
• Anteriorly: It passes over the infraclavicular fossa and deltopectoral groove to become continuous with the fascia
covering the deltoid.
• Superolaterally: The fascia curves around the inferolateral border of the pectoralis major to become continuous with
the axillary fascia.
• Inferolaterally: It is continuous with the fascia over the thorax and the rectus sheath.

1
Medico Express Block 2 GROSS ANATOMY
Key Structure of the Pectoral Region

PAST SEQ: Give origin, insertion, action, and nerve supply of muscles joining the upper limb to the thoracic wall in tabulated
form (supply 2016)

2
Medico Express Block 2 GROSS ANATOMY

SUMAMRY
Location & Function: The pectoral region is on the front of the chest, connecting the upper limb to the chest wall; it supports
limb movement, protection, and in females, lactation.
Key Structures: Includes the breast, superficial and deep fascia, cutaneous nerves and vessels, and the platysma muscle which
wrinkles the neck and protects the external jugular vein.
Major Muscles:
• Pectoralis Major: Adducts, flexes, and medially rotates the arm; originates from clavicle/sternum/ribs and inserts on
the humerus; supplied by medial and lateral pectoral nerves.
• Pectoralis Minor: Stabilizes and pulls scapula forward; arises from ribs 3–5 and inserts on the coracoid process;
supplied by the medial pectoral nerve.
Other Muscles:
• Subclavius: Depresses and stabilizes clavicle; from 1st rib to clavicle; supplied by nerve to subclavius.
• Serratus Anterior: Protracts and rotates scapula; from upper 8–9 ribs to medial scapula border; supplied by the long
thoracic nerve.

3
Medico Express Block 2 GROSS ANATOMY

MS-A-002 Human Anatomy (Dermatomes and cutaneious innervation of Upper Limb) Learning Objective
Describe the cutaneous nerves. and Superficial veins of the Upper Limb.
Describe the extent, attachments, and structures passing through Clavipectoral Fascia

CUTANEOUS NERVES OF THE UPPER LIMB


Cutaneous Nerve Contributing Source Course and Distribution
Spinal Nerves
Supraclavicular nerves C3, C4 Cervical plexus Pass anterior to the clavicle, immediately deep to
the platysma, and supply the skin over the clavicle
and superolateral aspect of the pectoralis major
The superior lateral C5, C6 Terminal branch of Emerges from beneath the posterior margin of the
cutaneous nerve of the axillary nerve deltoid and supplies skin over the lower part of this
arm muscle and on lateral side of the midarm
Inferior lateral C5, C6 Radial nerve (or posterior Perforates lateral head of triceps, passing close to
cutaneous nerve of arm cutaneous nerve of arm) the cephalic vein to supply skin over inferolateral
aspect of the arm
The posterior C5–C8 Radial nerve (in axilla) Crosses posterior to and communicates with
cutaneous nerve of arm intercostobrachial nerve and supplies skin on
posterior arm as far as olecranon
Posterior cutaneous C5–C8 Radial nerve (with inferior Perforates lateral head of triceps, descends
nerve of forearm lateral cutaneous nerve of laterally in arm, then runs along and supplies
arm) posterior forearm to wrist.
Lateral cutaneous C6–C7 Musculocutaneous nerve Emerges lateral to biceps tendon deep to the
nerve of forearm (terminal branch) cephalic vein, supplying the skin of anterolateral
forearm to the wrist
Medial cutaneous C8, T1 Medial cord of brachial Descends medial to brachial artery, pierces deep
nerve of forearm plexus (in axilla) fascia with a basilic vein in midarm, dividing into
anterior and posterior branches that enter forearm
and supply skin of anteromedial aspect to wrist
Medial cutaneous C8–T2 Medial cord of brachial Communicates with intercostobrachial nerve,
nerve of arm plexus (in axilla) continuing to supply the skin of medial aspect of
distal arm
Intercostobrachial T2 Second intercostal nerve Extends laterally, communicating with posterior
nerve (as its lateral cutaneous and medial cutaneous nerves of the arm, supplying
branch) skin of axilla and medial aspect of proximal arm
PAST SEQ: Draw and label a diagram to show cutaneous nerve supply of front and back of forearm and hand (supply 2020)

4
Medico Express Block 2 GROSS ANATOMY
PAST SEQ: DEFINDE dermatomes. Draw and Label dermatomes of upper limb (Annual 2015)
Q1. a) Name the cutaneous nerve of the forearm. b) Give the area of distribution. c) Illustrate your answer with diagram
(Annual 2007).
Dermatomes:
Dermatomes are specific areas of skin that are supplied by sensory nerve fibers from a single spinal nerve root.

Segmental (dermatomal) and peripheral (cutaneous nerve) innervation of the upper limb. A and B. The pattern of segmental
(dermatomal) innervation of the upper limb proposed by Foerster (1933) depicts innervation of the medial aspect of the limb by
upper thoracic (T1–T3) spinal cord segments, consistent with the experience of heart pain (angina pectoris) referred to that
area. C and D. The pattern of segmental innervation proposed by Keegan and Garrett (1948) has gained popular acceptance,
perhaps because of the regular progression of its stripes and correlation with developmental concepts. In both patterns, the
dermatomes progress sequentially around the periphery of the outstretched limb (with the thumb directed superiorly),
providing a way to approximate the segmental innervation.

SUPERFICIAL VEINS OF UPPER LIMB


Vein Origin Course Distribution Major Branches
Cephalic Vein Lateral side of Ascends along the Drains into the Anterior and
dorsal venous lateral side of the deltopectoral triangle posterior branches
network forearm and arm
Basilic Vein Medial side of Ascends along the Drains into the axillary Median cubital
dorsal venous medial side of the vein vein, median
network forearm and arm antebrachial vein
Median Anastomosis Runs anteriorly in the Connects cephalic and -
Cubital Vein between cephalic elbow region basilic veins
and basilic veins
Median Radial side of the Ascends along the Joins the basilic vein -
Antebrachial forearm radial side of the
Vein forearm

5
Medico Express Block 2 GROSS ANATOMY
Dorsal Venous Hand dorsal veins Spreads across the Drains into cephalic and -
Network back of the hand basilic veins

CLAVIPECTORAL FASCIA
Location:
• Fibrous sheet situated deep to the clavicular portion of the pectoralis major muscle.
Extent:
• Extends from the clavicle above to the axillary fascia below.
• Upper part splits to enclose the subclavius muscle.
• Posterior lamina is fused to the investing layer of the deep cervical fascia and the axillary sheath.
Attachment:
• Inferiorly: Encloses Pectoralis Minor Muscle
• Medially: External Intercostal Muscle of Upper Intercostal Spaces
• Laterally: Coracoid Process
Part:
• Costocoracoid ligament
• Costocoracoid membrane
• Suspensory ligament of axilla
Suspensory Ligament:
• Continues as the suspensory ligament below the muscle, attached to the dome of the axillary fascia.
Piercing Structures:
• Lateral Pectoral Nerve
• Cephalic Vein
• Thoracoacromial Vessels
• Lymphatics from Breast and Pectoral Region to Apical Group of Axillary Lymph Node
PAST SEQ: What is clavipectoral fascia? Name its parts and describe its attachments
Enumerate the structures passing through it

SUMMARY
Cutaneous Nerves of Upper Limb:
These nerves arise from cervical and brachial plexuses (C3–T2) and include supraclavicular, axillary, radial, musculocutaneous,
and intercostal nerves, supplying skin over the shoulder, arm, forearm, and axilla.
Superficial Veins of the Upper Limb:
Major veins include cephalic, basilic, median cubital, and median antebrachial, draining the upper limb into deeper veins;
commonly used for venipuncture and interconnected via the dorsal venous network.

6
Medico Express Block 2 GROSS ANATOMY
Clavipectoral Fascia – Structure & Extent:
A fibrous sheet deep to the pectoralis major, extending from the clavicle to the axillary fascia; encloses the subclavius and
pectoralis minor muscles, and blends with deep cervical and axillary fascia.
Clavipectoral Fascia – Attachments & Piercings:
Attached medially to intercostal muscles, laterally to the coracoid process, and forms the suspensory ligament of the axilla;
pierced by the lateral pectoral nerve, cephalic vein, thoracoacromial vessels, and lymphatics.

7
Medico Express Block 2 GROSS ANATOMY

MS-A-003 Human Anatomy/Integrate with Medicine (Pectoral region & Back) Learning Objectives
Define the boundaries of auscultation and state its clinical significance
Describe the osteology of the bones in pectoral region. Enumerate the superficial muscles of back, connecting
shoulder girdle with vertebral column.
Describe the 1. Attachments 2. Nerve supply Actions of Trapezius, Latissimus Dorsi, Rhomboid major and minor.
Mention the neurovascular supply of pectoral region and Correlate with important clinical conditions. Describe
superficial muscles of the back with their origin, insertion, nerve supply and actions.

TRIANGLE OF AUSCULTATION
Boundaries
The triangle of auscultation is a small triangular interval bounded as follows:
Boundaries Description
Medial Lateral border of the trapezius
Lateral Medial border of the scapula
Inferior Upper border of the latissimus dorsi
Floor 5th and 7th ribs, 6th intercostal space (ICS), and the rhomboid major

PAST SEQ: b) What is Triangle of Auscultation? Give its boundaries. (Annual 2019)
The chest of a patient was auscultated with arms abducted and a small area on back was located where the breath sounds were
easily and clearly audible. Name the area and it is limited by which structures? (Supply Exam 2024 held in 2025)

Clinical Description
Significance
Muscle The triangle in this area is the only part of the back not covered by large muscles, making it significant in
Coverage clinical examinations and diagnostics.
Respiratory Respiratory sounds from the apex of the lower lobe can be heard more clearly over this triangle on each
Sounds side, making it a valuable auscultation site for respiratory assessments with a stethoscope.
Esophageal On the left side, the cardiac orifice of the stomach lies deep to the triangle. Before the discovery of X-rays,
Tumor sounds of swallowed liquids were auscultated over this triangle to confirm esophageal tumors.
PAST SEQ: What is trainagle of ausculatation. Give its boundaries. (Annual 2019)
Bone Description
Clavicle - S-shaped bone connecting the arm to the trunk of the body.
- Articulates medially with the sternum at the sternoclavicular joint and laterally with the acromion process
of the scapula at the acromioclavicular joint.
- Provides attachment points for muscles such as deltoid, trapezius, and pectoralis major muscles.
- Supports the shoulder, aids in arm movement, and protects underlying neurovascular structures.

Scapula - Triangular-shaped bone located on the posterior aspect of the thoracic wall.
- Features include spine, acromion process, coracoid process, and glenoid cavity.
- Spine divides scapula into supraspinous and infraspinous fossae, providing attachment sites for muscles.
8
Medico Express Block 2 GROSS ANATOMY
- Acromion process articulates with the clavicle, forming the bony tip of the shoulder.
- Coracoid process serves as an attachment site for muscles and ligaments.
- Glenoid cavity articulates with the head of the humerus, forming the shoulder joint (glenohumeral joint).

Ribs - Long, curved bones forming the rib cage, protecting vital organs.
- Typically 12 pairs; first seven pairs are true ribs, next three pairs are false ribs, last two pairs are floating ribs.
- Each rib has a head, neck, tubercle, and shaft.
- Head articulates with thoracic vertebrae, forming costovertebral joints.
- Tubercle articulates with transverse process of the vertebra.

PAST SEQ: Q1. Name two muscles of the upper limb known as “climbing muscles”. Write their main attachments and nerve supply.
(Annual 2009) (Ans: Latissimus dorsi and pectoralis Major)

NEUROVASCULAR SUPPLY OF PECTORAL REGION


Cutaneous Nerves Area of Supply Nerve Origin
Medial, Intermediate, Supraclavicular Upper half of deltoid, clavicle down to
Nerves second rib Cervical plexus (C3, C4)
Anterior and Lateral Cutaneous
Branches Below second rib Second to sixth intercostal nerves
Floor of axilla, upper half of medial
Intercostobrachial Nerve (T2) side of arm T2 spinal nerve

Cutaneous Vessels Accompanying Structures Significance/Function


Anterior Cutaneous Perforating branches of the internal Supplying the anterior chest wall and
Nerves thoracic artery accompanying the nerves
Lateral cutaneous branches of the Supplying the lateral chest wall and
Lateral Cutaneous Nerves posterior intercostal arteries accompanying the nerves
In females, the second, third, and fourth perforating branches of the internal thoracic artery are notably larger to supply the
breast.
CLINICAL CORRELATES OF PECTORAL REGION

9
Medico Express Block 2 GROSS ANATOMY
Absence of Pectoral - Uncommon occurrence of absent part of pectoralis major muscle, typically affecting
Muscles sternocostal part.
- Limited disability; minimal impact.
- Physical changes: Absence of anterior axillary fold on affected side, lower position of the
nipple on affected side.
- Variation: Poland Syndrome, involving absence of both pectoralis major and minor
muscles, accompanied by breast hypoplasia and absence of two to four rib segments.
Effects of Paralyzed - Appearance of scapular wing due to lateral and posterior movement of scapula's medial
Serratus Anterior border.
- Deformation known as "winged scapula" when arm is raised; incomplete abduction of
upper limb due to inability to rotate glenoid cavity superiorly.
- Role of serratus anterior in rotating glenoid cavity hindered; trapezius aids in arm
elevation above horizontal.
Vulnerability of Long - Unique course on superficial aspect of serratus anterior, supplying it.
Thoracic Nerve - Increased vulnerability when limbs are elevated, exposing nerve to potential injury,
especially during knife fights or attacks.
Injury of Spinal - Clinical manifestation: Marked ipsilateral weakness observed during shoulder elevation
Accessory Nerve (CN XI) (shrugging) against resistance.
Injury of Thoracodorsal - Risk factors include surgical procedures in inferior axillary region, particularly during
Nerve mastectomies or scapular lymph node surgery.
- Functional impact of latissimus dorsi paralysis: Inability to raise trunk using upper limbs
or use axillary crutch; active depression of scapula necessary for certain activities.
- Paralysis affects rhomboid muscles, resulting in scapular positioning farther from midline
on affected side.
Injury to Dorsal Scapular - Paralysis affects rhomboid muscles, leading to scapular positioning farther from midline
Nerve compared to normal side.
Injury to Axillary Nerve - Damage leads to deltoid muscle atrophy, flattening shoulder contour and creating hollow
inferior to acromion.
- Loss of sensation possible over lateral side of proximal arm, supplied by superior lateral
cutaneous nerve of arm.
- Clinical implications include intramuscular drug injections using deltoid; awareness of
nerve location important during shoulder surgical approaches to prevent injury.
Rotator Cuff Injuries - Musculotendinous damage causing glenohumeral joint instability; tear or rupture of SITS
muscle tendons due to trauma or disease.
- Common injury: Rupture of supraspinatus tendon.
- Degenerative tendonitis prevalent in older individuals, leading to rotator cuff syndromes.

SUMMARY
Triangle of Auscultation
This triangular area, bounded by trapezius, scapula, and latissimus dorsi, lacks large muscle coverage, making it ideal for
auscultating respiratory sounds and historically used to detect esophageal issues.
Bones and Fascia of the Upper Body
The clavicle connects the upper limb to the trunk, the scapula supports shoulder motion, and the ribs protect thoracic organs.
The clavipectoral fascia encloses the pectoral muscles and allows passage of key neurovascular structures.
Neurovascular Supply of the Pectoral Region
Cutaneous nerves from cervical and intercostal sources innervate the skin of the chest and arm. Accompanying vessels, like
branches of the internal thoracic and intercostal arteries, supply the chest wall and breast.
Clinical Correlates of the Pectoral Region
Conditions include Poland syndrome (absent pectoral muscles), winged scapula (serratus anterior paralysis), and nerve
injuries (long thoracic, spinal accessory, thoracodorsal, axillary), each affecting shoulder stability, movement, or appearance.
Attachments & Nerve Supply:
• Trapezius: From skull and vertebrae (C7–T12) to clavicle, acromion, scapular spine; supplied by spinal accessory
nerve (motor) and C3–C4 (sensory).
• Latissimus Dorsi: From lower spine, iliac crest, ribs to humerus; supplied by thoracodorsal nerve (C6–C8).
• Rhomboids: From spinous processes (C7–T5) to medial scapula; supplied by dorsal scapular nerve (C4–C5).
Actions:
• Trapezius: Elevates, retracts, depresses, and upwardly rotates scapula.
• Latissimus Dorsi: Extends, adducts, and medially rotates the arm.
• Rhomboids: Retract and downwardly rotate scapula; stabilize it on thoracic wall.

10
Medico Express Block 2 GROSS ANATOMY
REVIEW POINTS
Arteries and Veins
1. Profunda brachii artery: Main artery of the posterior arm; ends as anterior and posterior descending branches.
2. Radial artery pulse: Felt lateral to flexor carpi radialis tendon at the wrist.
3. Cephalic vein: Begins from the lateral dorsal venous arch, pierces clavipectoral fascia, and crosses anatomical snuffbox.
4. Median cubital vein: Lies superficial to bicipital aponeurosis.
Muscles and Nerves
5. Thenar muscles (opponens pollicis, 1st & 2nd lumbricals): Supplied by the median nerve.
6. Remaining lumbricals and interossei: Supplied by the deep branch of the ulnar nerve.
7. Flexor pollicis longus: Supplied by anterior interosseous nerve.
8. Flexor digitorum superficialis: By median nerve.
9. Flexor carpi ulnaris: Supplied by ulnar nerve.
10. Extensor carpi ulnaris: By deep branch of radial nerve.
11. Flexor carpi radialis: Supplied by median nerve.
12. Biceps brachii, brachialis: Supplied by musculocutaneous nerve.
13. Supinator: Supplied by deep branch of radial nerve.
14. Subscapularis: Supplied by upper and lower subscapular nerves.
15. Pectoralis major: By medial and lateral pectoral nerves.
16. Pronator teres: Supplied only by median nerve.
Movements and Functions
17. Biceps brachii: Primary supinator when elbow is flexed.
18. Brachialis: Flexes the elbow.
19. Brachioradialis: Flexes forearm, especially in mid-prone position.
20. Flexor digitorum superficialis: Flexes middle phalanx.
21. Abduction of shoulder (0–180°): Requires intact upper trunk (C5, C6).
Clinical Correlations
22. Wrist drop: Caused by radial nerve palsy.
23. Colles' fracture: Distal radius fracture with posterior displacement — “dinner fork” deformity.
24. Dupuytren's contracture: Palmar fibromatosis causing finger flexion (esp. 4th & 5th digits).
25. Median nerve injury near flexor retinaculum: Leads to thenar muscle weakness and sensory loss.
26. Ulnar nerve injury: Inability to flex distal phalanges of 4th and 5th fingers (FDP involvement).
Anatomical Structures and Relations
27. Palmar aponeurosis: Tendinous extension of palmaris longus.
28. Radial bursa: Sheath of flexor pollicis longus, extending to thumb's distal phalanx.
29. Flexor retinaculum: Connects scaphoid/trapezium (laterally) to pisiform/hamate (medially).
30. Vincula tendina: Connect FDS & FDP to phalanges; carry tendon blood supply.
31. Space of Parona: Deep forearm space anterior to pronator quadratus.
32. Flexor digitorum superficialis/profundus + median nerve: Pass deep to flexor retinaculum.
33. Adductor pollicis: Supplied by deep ulnar nerve; middle finger lacks palmar interossei.
34. Abductor pollicis brevis: Supplied by median nerve, most lateral in thenar group.
35. Muscles of deep forearm: All except flexor carpi ulnaris.
36. Biceps brachii pierces deep fascia: Lateral to its tendon, becomes lateral cutaneous nerve of forearm.
Joints and Bones
37. Wrist joint: Ellipsoid type; involves radius, articular disc, scaphoid, lunate, triquetral.
38. Subscapularis inserts on lesser tubercle; other rotator cuff muscles attach to greater tubercle.
39. Carpal bones:
• Proximal row: Scaphoid, Lunate, Triquetral, Pisiform.
• Distal row: Trapezium, Trapezoid, Capitate, Hamate.
40. Supination > Pronation: Supination is stronger and more powerful.
Muscle Actions, Nerve Supply, and Clinical Testing
41. Opponens pollicis, involved in finger counting, is supplied by the median nerve.
42. Pronator teres, a forearm flexor, is innervated by the median nerve.
43. The long thoracic nerve supplies the serratus anterior; injury causes paralysis of the muscle and winging of the
scapula.
44. The axillary nerve arises from the posterior cord of the brachial plexus with root values C5–C6.
45. The deltoid, supplied by the axillary nerve, becomes paralyzed following its injury.
46. The supraspinatus muscle initiates abduction of the arm up to 15°.
47. Thumb abduction occurs at the first carpometacarpal joint.
48. The thenar muscles are primarily innervated by the median nerve.
49. Ulnar nerve injury results in paralysis of the third and fourth lumbricals, leading to a claw hand.
112
Medico Express Block 2 GROSS ANATOMY

PAST MCQS
1. A person falls from the roof and sustains a fracture of the shaft specifically segments C5-C6 or at some point above it in the
of the humerus. Which of the following nerve injuries occurs: spinal cord or brain.
a) Musculocutaneous nerve 7. A student was clinically diagnosed as having a pinch deformity,
b) Ulnar nerve which is the loss of opposition of the thumb. Which nerve
c) Median nerve injury occurs:
d) Radial nerve a) Subscapular nerve
e) Nerve to subclavius b) Axillary nerve
Correct Answer: d) Radial nerve c) Median nerve
A fracture of the shaft of the humerus can damage the radial nerve, d) Radial nerve
which is commonly affected in such injuries. e) Ulnar nerve
2. Loss of supination of the forearm occurs due to loss of action Correct Answer: c) Median nerve
of which of the following muscles Loss of opposition of the thumb is often associated with damage to
a) Deltoid the median nerve.
b) Subscapularis 8. Which of the following muscles is supplied by the
c) Triceps brachii musculocutaneous and radial nerves:
d) Biceps brachii a) Brachialis
e) Infraspinatus b) Coracobrachialis
Correct Answer: d) Biceps brachii c) Triceps brachii
The biceps brachii muscle plays a crucial role in the supination of d) Biceps brachii
the forearm. e) Deltoid
3. Loss of action of supination and flexion is due to the loss of Correct Answer: a) Brachialis
nerve, which is: Brachialis is primarily supplied by the musculocutaneous nerve,
a) Musculocutaneous nerve with a small lateral portion innervated by the radial nerve.
b) Axillary nerve 9. Which of the following lymph nodes drain the lateral thoracic
c) Radial nerve wall:
d) Median nerve a) Helical lymph
e) Ulnar nerve b) Lateral lymph
Correct Answer: a) Musculocutaneous nerve c) Posterior lymph
It occurs in musculocutaneous palsy; the nerve is damaged during d) Anterior lymph
penetrating trauma to the arm. Symptoms include an inability e) Medial lymph
to flex the elbow with the biceps brachii and weakened Correct Answer: d) Anterior lymph
supination, loss of sensation in the skin on the front side of the The anterior lymph nodes drain the lateral thoracic wall.
forearm, and weakened rotation of the arm. 10. Thickening of the anterior brachial fascia occurs in:
4. A patient was diagnosed with wrist drop, which occurs due to a) Extensor retinaculum
which of the following injuries: b) Flexor retinaculum
a) Ulnar nerve c) Deep fascial
b) Median nerve d) Superficial fascia
c) Radial nerve e) Peroneal retinaculum
d) Axillary nerve Correct Answer: a) Extensor retinaculum
e) Pectoral nerve Thickening of the anterior brachial fascia is known as the extensor
Correct Answer: c) Radial nerve retinaculum.
Wrist drop is a condition associated with radial nerve injury, leading 11. A 7-year-old boy falls on an outstretched hand with weight
to the inability to extend the wrist and fingers. landing on the palm. Which of the following bones is fractured:
5) Which of the following occurs in Erb's palsy: a) Lunate
a) The elbow is adducted and extended b) Trapezoid
b) Elbow abducted and extended c) Trapezium
c) Elbow abducted and rotated laterally d) Scaphoid
d) Elbow abducted and rotated medially e) Hamate
e) None of the above Correct Answer: d) Scaphoid
Correct Answer: a) Elbow is adducted and extended In such an injury, a fracture of the scaphoid bone is common.
Erb's palsy is characterized by arm weakness and loss of motion. It Symptoms of a scaphoid fracture typically include pain and
results from an injury to the 5th and 6th cervical nerves. The tenderness in the area just below the base of the thumb.
arm is adducted, and the forearm is extended, internally 12. Latissimus dorsi, which is a large and flat muscle, is present in
rotated, and pronated, generating the classical "waiter's tip" which of the following areas:
appearance. a) Axilla
6. The biceps tendon jerk is at the level of which of the following: b) Anterior wall of axilla
a) C5-C6 c) Posterior wall of axilla
b) C6-C7 d) Lateral to axilla
c) C8-T1 e) Medial to axilla
d) C3-C4 Correct Answer: c) Posterior wall of the axilla
e) C7-C8 The latissimus dorsi is a large, flat muscle located on the back and
Correct Answer: a) C5-C6. A change in the biceps reflex indicates stretches to the sides, behind the arm, and is partly covered
pathology at the level of the musculocutaneous nerve, by the trapezius on the posterior aspect near the midline.
13. The radial pulse can be felt at which of the following muscles:
115
Medico Express Block 2 GROSS ANATOMY
a) Flexor carpi radialis a) Extensor pollicis brevis
b) Flexor carpi ulnaris b) Extensor pollicis longus
c) Flexor digitorum c) Flexor pollicis longus
d) Peroneus longus d) Flexor carpi radialis
e) Peroneus Tertius e) Abductor pollicis longus
Correct Answer: a) Flexor carpi radialis Correct Answer: a) Extensor pollicis brevis
The radial pulse is typically felt between the wrist bone and the The medial wall of the anatomical snuffbox is formed by the tendon
tendon on the thumb side of the wrist, along the flexor carpi of the extensor pollicis brevis
radialis. 20. All of the following structures are located at the proximal end
14. Which of the following nerves is the adductor of digits: of the humerus except:
a) Ulnar nerve a) Lesser tubercle
b) Radial nerve b) Anatomical neck
c) Axillary nerve c) Surgical neck
d) Median nerve d) Head
e) Musculocutaneous nerve e) Olecranon process
Correct Answer: a) Ulnar nerve Correct Answer: e) Olecranon process
The adductor pollicis muscle is innervated by the deep branch of The olecranon process is located at the distal end of the ulna, not
the ulnar nerve. the proximal end of the humerus.
15. A 52-year-old patient is suffering from adhesive capsulitis; due 21. The tendons of the following muscles form the rotator cuff
to this, which abnormality can occur: except:
a) Carpal tunnel syndrome a) Teres minor
b) Ulnar claw hand b) Supraspinatus
c) Dupuytren's contracture c) Teres major
d) Frozen shoulder d) Infraspinatus
e) Both b and c e) Subscapularis
Correct Answer: d) Frozen shoulder Correct Answer: c) Teres major
Adhesive capsulitis is commonly referred to as "frozen shoulder," The rotator cuff is formed by the tendons of supraspinatus, teres
which results in immobility of the shoulder joint. It may occur minor, infraspinatus, and subscapularis, but not teres major.
during the recovery from a shoulder injury, broken arm, or a 22. Injury to the ulnar nerve results in the absence of sensation in
stroke. which part of the hand:
16. The lateral intermuscular septum is pierced by which of the a) Thumb
following vessels: b) Little finger
a) Brachial artery c) Index finger
b) Ulnar nerve d) Middle finger
c) Radial nerve e) Ring finger
d) Median nerve Correct Answer: b) Little finger
e) Axillary artery The little finger (pinky finger) is innervated by the ulnar nerve.
Correct Answer: c) Radial nerve Injury to the ulnar nerve can result in the loss of sensation in
The radial nerve originates from several nerve roots that run from the little finger.
the C5-C8 cervical vertebrae, making it an important structure 23. Regarding joints, which of the following joints is less stable but
in the lateral intermuscular septum. more mobile:
17. The combined movements of flexion, adduction, and medial a) Wrist joint
rotation are a function of which of the following muscles: b) Hip joint
a) Pectoralis minor c) Elbow joint
b) Teres minor d) Knee joint
c) Teres major e) Shoulder joint
d) Rhomboid minor Correct Answer: e) Shoulder joint
e) Pectoralis major The shoulder joint is a synovial joint that is less stable but allows for
Correct Answer: e) Pectoralis major greater mobility and multiaxial movement compared to other
The pectoralis major muscle extends across the upper part of the joints.
chest and is attached to the rear ridge of the humerus. It is 24. Injury to the anatomical snuffbox produces massive bleeding
responsible for actions such as arm adduction and rotation of due to damage to:
the arm forward about the body's axis. a) Brachial artery
18. Tenderness and inflammation of the palmar aponeurosis are b) Radial artery
known as which of the following: c) Ulnar artery
a) Carpal tunnel syndrome d) Common interosseous artery
b) Ulnar claw hand e) Metacarpal artery
c) Dupuytren's contracture Correct Answer: b) Radial artery
d) Volkmann ischemic contracture The anatomical snuffbox is a triangular depression on the hand's
e) All of the above dorsum, and injury to it can lead to massive bleeding due to
Correct Answer: c) Dupuytren's contracture damage to the radial artery.
Dupuytren's contracture is a condition characterized by 25. Which of the following muscles inserts into the pisiform and
tenderness, inflammation, and the development of a thick hook of hamate:
cord in the palm that can pull one or more fingers into a bent a) Palmaris longus
position over time. b) Flexor digitorm profundus
19. The medial wall of the anatomical snuffbox consists of the c) Flexor digitorum superficialis
tendon from: d) Flexor carpi ulnaris

116
Medico Express Block 2 GROSS ANATOMY

LOWER LIMB
MS-A-030 Human Anatomy (Hip Bone) Learning Objectives
Draw and label the Parts of the hip bone, with its attachments.
Describe the parts, attachments of hip bone
Identify the parts and bony features of the ho bone, with its attachments, important relations
Demonstrate the side determination of hip bone, its bony features, attachments
Side determination of hip bone:
Acetabulum Directed laterally
The flat, expanded ilium forming upper part Above the acetabulum.
of bone
Obturator foramen below the acetabulum, bounded anteriorly by pubis, and
posteriorly ischium
Parts:
This large irregular bone is made up of three parts. These are:
The ilium Superiorly
The pubis Anteroinferiorly
The ischium Posteroinferiorly
ILIUM
The ilium or flank has the following:
An upper end The iliac crest
A lower end Smaller, and fused with pubis and ischium at the acetabulum
Three borders anterior
posterior
medial
Three surfaces gluteal surface
iliac surface/ iliac fossa
sacropelvic surface.
• ilium forms the upper two-fifths of the acetabulum.
These parts are described one by one below.
➢ Iliac Crest:
• The iliac crest is a broad convex ridge forming the upper end of the ilium.
• It can be felt in the living at the lower limit of the flank.
• Curvatures:
Vertically it is convex upwards, anteroposteriorly, it is concave inwards in front and concave outwards behind.
• The highest point of the iliac crest
situated a little behind the midpoint of the crest.
It lies at the level of the disc between the spines of vertebrae L3 and L4.
• Ends:
The anterior end of the iliac crest is called the anterior superior iliac spine. This is a prominent landmark that is easily
felt in the living.
• The posterior end of the crest is called the posterior superior iliac spine. Its position on the surface of the body is
marked by a dimple 4 cm lateral of the second sacral spine.
• Morphological divisions.
the iliac crest is divided into a long ventral segment and a short dorsal segment.
• The ventral segment forms more than the anterior two-thirds of the crest.
• It has an outer lip, an inner lip, and an intermediate area. EXPRESS HIT
• The tubercle of the iliac crest is an elevation that lies on the outer lip about
5 cm behind the anterior superior iliac spine. ASIS serves as a landmark for spinal
• The dorsal segment forms less than the posterior one-third of the crest. anesthesia at the level of L3–L4
• It has a lateral and a medial slope separated by a ridge. interspace, aligning with the highest
➢ Anterior Border of Ilium: point of the iliac crest.
• This border starts at the anterior superior iliac spine and runs downwards
to the acetabulum.

132
Medico Express Block 2 GROSS ANATOMY
• The upper part of the border presents a notch, while its lower part shows an elevated area called the anterior inferior
iliac spine.
• The lower half of this spine is large, triangular and rough.
➢ Posterior Border of Ilium:
• This border extends from the posterior superior iliac spine to the upper end of the posterior border of the ischium.
• A few centimeters below the posterior superior iliac spine it presents another prominence called the posterior inferior
iliac spine.
• Still lower down the posterior border is marked by a large deep notch called the greater sciatic notch.
➢ Medial Border:
• It extends on the inner or pelvic surface of the ilium from the iliac crest to the iliopubic eminence.
• It separates the iliac fossa from the sacropelvic surface.
• Its lower rounded part forms the iliac parts of the arcuate line or inlet of pelvis.
➢ Gluteal Surface:
• This is the outer surface of the ilium, which is convex in front and concave behind, like the iliac crest.
• It is divided into four areas by three gluteal lines.
• The posterior gluteal line, the shortest, begins 5 cm in front of the posterior superior spine, and ends just in front of the
posterior inferior spine.
• The anterior gluteal line, the longest, begins about 2.5 cm behind the anterior superior spine, runs backwards and then
downwards to end at the middle of the upper border of the greater sciatic notch.
• The inferior gluteal line, the most ill-defined, begins a little above and behind the anterior inferior spine, runs
backwards and downwards to end near the apex of the greater sciatic notch.
➢ Iliac Fossa:
• This is the large concave area on the inner surface of the ilium, situated in front of its medial border.
• It forms the lateral wall of the false pelvis.
➢ Sacropelvic Surface:
• This is the uneven area on the inner surface of the ilium, situated behind its medial border.
• It is subdivided into three parts; the iliac tuberosity, the auricular surface and the pelvic surface.
• The iliac tuberosity is the upper, large, roughened area, lying just below the dorsal segment of the iliac crest. It is raised
in the middle and depressed both above and below.
• The auricular surface is articular but pitted. It lies anteroinferior to the iliac tuberosity. It articulates with the sacrum to
form the sacroiliac joint.
• The pelvic surface is smooth and lies anteroinferior to the auricular surface.
• It forms a part of the lateral wall of the true pelvis. Along the upper border of the greater sciatic notch, this surface is
marked by the preauricular sulcus.
• This sulcus is deeper in females than in males.
PUBIS:
It has:
(1) A body Anteriorly
(2) A superior ramus Superolaterally
(3) An inferior ramus Inferolaterally
Body of Pubis:
This is flattened from before backwards, and has:
(1) A superior border The pubic crest.
(2) A pubic tubercle At the lateral end of the pubic crest.
(3) Three surfaces • Anterior,
• Posterior
• Medial.
• The pubic tubercle at the lateral end of the pubic crest forms an important landmark.
• The anterior surface is directed downwards, forwards and slightly lateral. It is rough supero-medially and smooth
elsewhere.
• The posterior or pelvic surface is smooth. It is directed upwards and backwards. It forms the anterior wall of the true
pelvis, and is related to the urinary bladder.
• The medial or symphyseal surface articulates with the opposite pubis to form pubic symphysis.
Superior Ramus:
Extent From the body of the pubis to acetabulum
Three • The superior border is called the pectineal line or pecten pubis.
borders: • It is a sharp crest extending from just behind the pubic tubercle to the posterior part of the iliopubic
eminence.

133
Medico Express Block 2 GROSS ANATOMY
• With the pubic crest it forms the pubic part of the arcuate line.
• The anterior border is called the obturator crest. The border is a rounded ridge, extending from the
pubic tubercle to the acetabular notch.
• The inferior border is sharp and forms the upper margin of the obturator foramen.
Three • The pectineal surface is a triangular area between the anterior and superior borders, extending from the
surfaces: pubic tubercle to the iliopubic eminence.
• The pelvic surface lies between the superior and inferior borders.
It is smooth and is continuous with the pelvic surface of the body of the pubis.
• The obturator surface lies between the anterior and inferior borders.
• It presents the obturator groove.
Inferior Ramus:
• Extents from body of pubis to ramus of the ischium, medial to the obturator foramen.
• It unites with the ramus of the ischium to form the conjoined ischiopubic rami.
ISCHIUM:
Body of the Ischium:
This is a thick and massive mass of bone that lies below and behind the acetabulum. It has:
Two ends, Upper • The upper end forms the posteroinferior two-fifths of the acetabulum.
• The ischium, ilium and pubis fuse with each other in the acetabulum.
Lower • The lower end forms the ischial tuberosity.
• It gives off the ramus of the ischium which forms an acute angle with the body
Three Anterior • The anterior border forms the posterior margin of the obturator foramen.
borders, Posterior • The posterior border is continuous above with the posterior border of the ilium.
• Below, it ends at the upper end of the ischial tuberosity.
• It forms part of the lower border of ilium.
• It also forms part of the lower border of the greater sciatic notch.
• Below the spine the posterior margin shows a projection called the ischial spine.
• Below the spine the posterior border shows a concavity called the lesser sciatic notch.
Lateral • The lateral border forms the lateral margin of the ischial tuberosity, except at the upper
end where it is rounded.
Three Femoral • The femoral surface lies between the anterior and lateral borders.
surfaces, Dorsal • The dorsal surface is continuous above with the gluteal surface of the ilium.
• From above downwards it presents a convex surface adjoining the acetabulum, a wide
shallow groove, and the upper part of the ischial tuberosity.
• The ischial tuberosity is divided by a transverse ridge into an upper and a lower area.
• The upper area is subdivided by an oblique ridge into a superolateral area and an
inferomedial area.
• The lower area is subdivided by a longitudinal ridge into outer and inner area.
Pelvic • The pelvic surface is smooth and forms part of the lateral wall of the true pelvis.
Conjoined Ischiopubic Rami:
The inferior ramus of the pubis unites with the ramus of the ischium on the medial side of the obturator foramen. The site of
union may be marked by a localized thickening. The conjoined rami have:
Two borders Upper Forms part of the margin of the obturator foramen.
Lower Forms the pubic arch along with the corresponding border of the bone
of the opposite side.
Two surfaces Outer Rough and irregular; gives attachment to muscles such as the obturator
externus and adductor muscles of the thigh.
Inner The inner surface is convex and smooth. It is divided into three areas,
upper, middle and lower, by two ridges

134
Medico Express Block 2 GROSS ANATOMY

ATTACHMENTS ON HIP BONE:


Attachments on the ilium:
Point on the hip bone. Attachment with the description.
Anteriorsuperioriliac • Lateral end of the inguinal ligament.
spine: • Origin to the sartorius muscle.
The outer lip of the iliac • Attachment to the fascia lata in its whole extent;
crest: • Origin to the tensor fasciae latae in front of the tubercle;
• Insertion to the external oblique muscle in its anterior two-thirds;
• Origin to the latissimus dorsi just behind the highest point of the crest.
• The tubercle of the crest marks the point of maximum traction by the
iliotibial tract.
The inner lip of the iliac • Origin to the transversus abdominis in its anterior two-thirds;
crest: • Attachment to the fascia transversalis and to the fascia iliaca in its anterior
two-thirds, deep to the attachment of the transversus abdominis;
• Origin to the quadratus lumborum in its posterior one-third; and
• Attachment to the thoracolumbar fascia around the attachment of the
quadratus lumborum.
Intermediate area of the • Origin to the internal oblique muscle in its anterior two-thirds.
iliac crest:
Dorsal segment of the • The lateral slope gives origin to the gluteus maximus.
iliac crest: • The medial slope gives origin to the erector spinae.
• The interosseous and dorsal sacroiliac ligaments are attached to the medial
margin deep to the attachment of the erector spinae.
Upper half of the • Provides origin to the straight head of the rectus femoris muscle. The lower
anterior inferior iliac half gives attachment to the iliofemoral ligament, which is part of the hip
spine: joint capsule.
Lower part of this spine:
Posterior border of the • Attachment to the upper fibres of the sacrotuberous ligament above the
ilium: greater sciatic notch; and
• Origin to a few fibres of the piriformis from the upper margin of the greater
sciatic notch.
Gluteal surface: • The area behind the posterior gluteal line gives origin to the upper fibres of
the gluteus maximus.
• The gluteus medius arises from the area between the anterior and posterior
gluteal lines,
• The gluteus minimus arises from the area between the anterior and inferior
gluteal lines,
135
Medico Express Block 2 GROSS ANATOMY
• Below the inferior gluteal line, the reflected head of the rectus femoris arises
from the groove above the acetabulum
• The capsular ligament of the hip joint is attached along the margin of the
acetabulum.
Iliac fossa: • Origin to the iliacus from its upper two-thirds.
Iliac tuberosity: • The interosseous sacroiliac ligament in its greater part,
• The dorsal sacroiliac ligament posteriorly, and
• The iliolumbar ligament superiorly.
Convex margin of the • The ventral sacroiliac ligament.
auricular surface:
Pelvic surface: • The preauricular sulcus provides attachment to the lower fibres of the
ventral sacroiliac ligament.
• The part of the pelvic surface lateral to the preauricular sulcus gives origin to
a few fibres of the piriformis,
• The rest of the pelvic surface gives origin to the upper half of the obturator
internus.

Attachments and relations of the pubis:


Point on the hip bone. Attachment with the description.
Pubic tubercle: • Medial end of the inguinal ligament.
• Ascending loops of the cremaster muscle.
• In males, the tubercle is crossed by the spermatic cord.
Pubic crest: • Medial part is crossed by the medial head of the rectus abdominis.
• Lateral part gives origin to:
• The lateral head of the rectus abdominis,
• And to the pyramidalis.
Anterior surface of the body of • Attachment to the anterior pubic ligament medially,
the pubis: • Origin to the adductor longus in the angle between the crest and the symphysis,
• Origin to the gracilis, from the margin of the symphysis, and from the inferior
ramus,
• Origin to the adductor brevis lateral to the origin of the gracilis, and
• Origin to the obturator externus near the margin of the obturator foramen.
Posterior surface of the body of • Origin to the levator ani from its middle part,
the pubis: • Origin to the obturator internus laterally, and
• Attachment to the puboprostatic ligaments medial to the attachment of the levator
ani.
Pectineal line: • The conjoint tendon at the medial end,
• The lacunar ligament at the medial end, in front of the attachment of the conjoint
tendon;
• The pectinate ligament along the whole length of the line lateral to the attachment
of the lacunar ligament,
• The pectineus muscle which arises from the whole length of the line,
• The fascia covering the pectineus, and
• the psoas minor, which is inserted here when present.
Upper part of the pectineal • Origin to the pectineus.
surface:
Pelvic surface: Crossed by:
• The ductus deferens in males, and
• The round ligament of the uterus in females.
Obturator groove: • Transmits the obturator vessels and nerve.
Attachments and relations of the ischium:
Point on the hip Attachment with the description.
bone.
Ischial spine: • sacrospinous ligament along its margins
• origin for the posterior fibres of the levator ani from its pelvic surface.
• Its dorsal surface is crossed by the internal pudendal vessels and by the nerve to the
obturator internus.
Lesser sciatic notch: • occupied by the tendon of the obturator internus.
136
Medico Express Block 2 GROSS ANATOMY
• The upper and lower margins of the notch give origin to the superior and inferior gemelli
respectively
Femoral surface of • the obturator externus along the margin of the obturator foramen
the ischium: • the quadratus femoris along the lateral border of the upper part of the ischial tuberosity.
Dorsal surface of • The upper convex area is related to the piriformis, the sciatic nerve, and the nerve to the
the ischium: quadratus femoris.
• The groove transmits the tendon of the obturator internus.
Ischial tuberosity: • The superolateral area gives origin to the semimembranosus,
• the inferomedial area to the semitendinosus and the long head of the biceps femoris,
• the outer lower area to the adductor magnus.
• The inner lower area is covered with fibrofatty tissue which supports body weight in the sitting
position. The sharp medial margin of the tuberosity gives attachment to the sacrotuberous
ligament.
• The lateral border of the ischial tuberosity provides attachment to the ischiofemoral ligament, just
below the acetabulum.
Pelvic surface: • The greater part gives origin to the obturator internus.
• The lower end of this surface forms the lateral wall of the ischioanal fossa.
Conjoined ➢ The upper border gives attachment to the obturator membrane.
ischiopubic rami: ➢ The lower border provides attachment to the fascia lata, and to the membranous layer of
superficial fascia or Colles' fascia of the perineum.
➢ The muscles taking origin from the outer surface are:
• the obturator externus, near the obturator margin of both rami
• the adductor brevis, chiefly from the pubic ramus;
• the gracilis, chiefly from the pubic ramus; and
• the adductor magnus, chiefly from the ischial ramus.
➢ The attachments on the inner surface are:
• The upper ridge gives attachment to the upper layer of the urogenital diaphragm.
• The perineal membrane is attached to the lower ridge.
➢ The upper area gives origin to the obturator internus.
➢ The middle area gives origin to the sphincter urethrae and to the deep transverse perinei,
• and is related to the dorsal nerve of the penis, and to the internal pudendal vessels.
➢ The lower area provides attachment to the crus penis, and
• gives origin to the ischiacavernosus and to the superficial transverse perinei.

137
Medico Express Block 2 GROSS ANATOMY

Sex Differences:
Characteristics In Male In Female
Curvature of iliac crest More pronounced Less pronounced
Depth of iliac fossa More less
Acetabulum Large Small
Greater sciatic notch Narrow Wider
Pubic crest Shorter Larger
138
Medico Express Block 2 GROSS ANATOMY
Periauricular sulcus Less marked More marked
Subpubic angle Less More
Obturator foramen Large, oval Small, triangular

Ossification:
The hip bone ossifies in cartilage from:
Three primary The primary centres appear: One for the ilium: During the second month of intrauterine
centres. life
One for the During the fourth month
ischium:
One for the pubis: During the fifth month
Five secondary Secondary centres appear at Two for the iliac crest,
centres. puberty Two for the Y-shaped cartilage of the acetabulum
One for the ischial tuberosity
➢ At birth the hip bone is ossified except for three cartilaginous parts. These are:
• The iliac crest.
• A y-shaped cartilage separating the ilium, ischium and pubis; and
• A strip along the inferior margin of the bone including the ischial tuberosity.
• The ischiopubic rami fuse with each other at 7 to 8 years of age.
• Ossification in the acetabulum is completed at 16-17 years, and the rest of the bone is ossified by 20-25 years.
• The anterior superior iliac spine, pubic tubercle and crest and the symphyseal surface may have separate
secondary centres of ossification.

SUMMARY
1. Parts and Side Determination of the Hip Bone:
• The hip bone consists of three fused parts: ilium (superior), pubis (anteroinferior), and ischium
(posteroinferior).
• Side determination is done by the position of the acetabulum (lateral), ilium (superior), and obturator
foramen (inferior).
2. Ilium – Features and Attachments:
• Major parts: iliac crest, anterior/posterior/medial borders, gluteal and iliac surfaces.
• Important landmarks: ASIS, PSIS, iliac crest tubercle.
• Muscle attachments include: gluteus maximus/medius/minimus, iliacus, sartorius, quadratus lumborum,
abdominal muscles, and erector spinae.
• Articulates with the sacrum at the auricular surface forming the sacroiliac joint.
3. Pubis – Features and Attachments:
• Has body, superior ramus, and inferior ramus.
• Key landmarks: pubic crest, pubic tubercle, pectineal line.
• Muscle attachments include: rectus abdominis, pyramidalis, adductor longus, brevis, gracilis, obturator
externus/internus, and levator ani.
• Forms anterior wall of the pelvis and contributes to pubic symphysis.

139
Medico Express Block 2 GROSS ANATOMY
4. Ischium – Features and Attachments:
• Has body and ramus, forming posteroinferior part of acetabulum and ischial tuberosity.
• Key landmarks: ischial spine, greater and lesser sciatic notches.
• Muscle attachments: hamstrings, quadratus femoris, obturator internus, gluteus maximus.
• Provides support during sitting through the ischial tuberosity.
5. Conjoined Ischiopubic Rami and Clinical Relevance:
• Formed by the union of inferior pubic ramus and ischial ramus, enclosing the obturator foramen.
• Important in forming the pubic arch and related to muscle and ligament attachments.
• Clinical landmarks like ASIS, PSIS, pubic tubercle, and iliac crest are palpated for surface anatomy and
procedures like lumbar puncture or bone grafting.

140
Medico Express Block 2 GROSS ANATOMY

MS-A-031 Human Anatomy (Femur) Learning Objectives


Describe the parts, attachments, side determinatin of femur
Identify the parts and bony features of the femur, with its attachments.
Demonstrate the side determination of femur, its bony features, attachments, and important relations (correlate
these with fractures)
Describe coax Vara and coax valga and their clinical significance.
FEMUR:
• The femur or thigh bone is the longest and the strongest bone of the body.
Side determination:
The upper end: Bears a rounded head
The lower end: Widely expanded to form two large condyles.
The head: Directed medially.
The cylindrical shaft: Convex forwards.
Parts:
Like any other long bone, it has
o Two ends upper and lower, and
o A shaft.
Upper End
The upper end of the femur includes the head, the neck, the greater trochanter, the lesser trochanter, the intertrochanteric line,
and the intertrochanteric crest.
Head: • The head forms more than half a sphere, and is directed medially, upwards and
slightly forwards.
• It articulates with the acetabulum to form the hip joint.
• A roughened pit is situated just below and behind the centre of the head. This pit is
called the fovea.
Neck: • It connects the head with the shaft and is about 3.7 cm long.
➢ It makes an angle with the shaft. The neck-shaft angle is about 125° in adults. It is
less in females due to their wider pelvis.
• The angle facilitates movements of the hip joint.
• It is strengthened by a thickening of bone called the calcar femorale present along
its concavity.
➢ The neck has two borders and two surfaces.
The upper border • Concave and horizontal,
• Meets the shaft at the greater trochanter.
The lower border, • Straight and oblique,
• Meets the shaft near the lesser trochanter.
The anterior • Flat.
surface • Meets the shaft at the intertrochanteric line.
• It is entirely intracapsular.
• The articular cartilage of the head may extend to this surface.
The posterior • Convex from above downwards
surface • And concave from side to side.
• Meets the shaft at the intertrochanteric crest.
• Only a little more than its medial half is intracapsular.
• It is crossed by a horizontal groove for the tendon of the
obturator externus.
➢ The angle of femoral torsion or angle of ante-version is formed between the
transverse axes of the upper and lower ends of the femur.
➢ It is about 15 degrees.
Greater Trochanter: • This is large quadrangular prominence located at the upper part of the junction of
neck with the shaft.
• The greater trochanter has an upper border with an apex, three surfaces, anterior,
medial and lateral.
Lesser Trochanter: • It is a conical eminence directed medially and backwards from the junction of the
posteroinferior part of the neck with the shaft.
Intertrochanteric Line: • Marks the junction of the anterior surface of the neck with the shaft of the femur.

141
Medico Express Block 2 GROSS ANATOMY
• It is a prominent roughened ridge which begins above, at the anterosuperior angle
of the greater trochanter as a tubercle,
• And is continuous below with the spiral line in front of the lesser trochanter.
Intertrochanteric • Marks the junction of the posterior surface of the neck with the shaft of the femur.
Crest: • It is a smooth-rounded ridge, which begins above at the postero-superior angle of
the greater trochanter and ends at the lesser trochanter.
• The rounded elevation, a little above its middle, is called the quadrate tubercle.
Shaft: • The shaft is more or less cylindrical.
• It is narrowest in the middle,
• More expanded inferiorly than superiorly.
• It is convex forwards and is directed obliquely downwards and medially,
• Because the upper ends of two femora are separated by the width of the pelvis,
and their lower ends are close together.
• In the middle one-third, the shaft has three borders, medial, lateral and posterior
and three surfaces, anterior, medial and lateral.
• The medial and lateral borders are rounded and ill-defined, but the posterior
border is in the form of a broad roughened ridge, called the linea aspera.
• The linea aspera has distinct medial and lateral lips.
• The medial and lateral surfaces are directed more backwards than towards the
sides.
Lower End: • The lower end of the femur is widely expanded to form two large condyles, one
medial and one lateral.
• Anteriorly, the two condyles are united and are in line with the front of the shaft.
• Posteriorly, they are separated by a deep gap, termed the intercondylar fossa or
intercondylar notch, and project backwards much beyond the plane of the popliteal
surface.
Articular Surface: • The two condyles are partially covered by a large articular surface which is divisible
into patellar and tibial parts.
• The patellar surface covers the anterior surfaces of both condyles, and extends
more on the lateral condyle than on the medial. It is separated from the tibial
surfaces by two faint grooves.
• The tibial surfaces cover the inferior and posterior surfaces of the two condyles,
and merge anteriorly with the patellar surface.
Lateral Condyle: • The lateral condyle is flat laterally, and is more in line with the shaft.
• It, therefore, takes greater part in the transmission of body weight to the tibia.
• Though it is less prominent than the medial condyle, it is stouter and stronger. The
lateral aspect presents the following:
(a) A prominence called the lateral epicondyle
(b) The popliteal groove which lies just below the epicondyle. It has a deeper
anterior part and a shallower posterior part.
(c) A muscular impression posterosuperior to the epicondyle.

Medial Condyle: • This condyle is convex medially.


• The most prominent point on it is called the medial epicondyle.
• Posterosuperior to the epicondyle there is a projection, the adductor tubercle. This
tubercle is an important landmark.
• The epiphyseal line for the lower end of the femur passes through it.
Intercondylar Fossa or • This notch separates the lower and posterior parts of the two condyles.
Intercondylar Notch: • It is limited anteriorly by the patellar articular surface,
• And posteriorly by the intercondylar line which separates the notch from the
popliteal surface.

142
Medico Express Block 2 GROSS ANATOMY

ATTACHMENTS:
Point on the femur Attachment with discription
Fovea on the head of • Ligament of the head of femur or round ligament, or ligamentum teres .
the femur
Greater trochanter. • The piriformis is inserted into the apex.
• The gluteus minimus is inserted into the rough lateral part of the anterior surface.
• The obturator internus and the two gemelli are inserted into the upper rough impression on the
medial surface.
• The obturator externus is inserted into the trochanteric fossa.
• The gluteus medius is inserted into the ridge on the lateral surface.
• The trochanteric bursa of the gluteus medius lies in front of the ridge,
• And the trochanteric bursa of the gluteus maximum lies behind the ridge.
Lesser trochanter • The psoas major is inserted on the apex and medial part of the rough anterior surface.
• The iliacus is inserted on the anterior surface of the base of the trochanter and on the area
below it.
• The smooth posterior surface of the lesser trochanter is covered by a bursa that lies deep to the
upper horizontal fibres of the adductor magnus.
Intertrochanteric line • The capsular ligament of the hip joint;
143
Medico Express Block 2 GROSS ANATOMY
• Upper band of the iliofemoral ligament in its upper part;
• Lower band of the iliofemoral ligament in its lower part;
• Origin to the highest fibres of the vastus lateralis from the upper end; and
• Origin to the highest fibres of the vastus medialis from the lower end of the line.
Quadrate tubercle • Insertion of the quadratus femoris.
Shaft • The medial and popliteal surfaces are bare, except for a little extension of the origin of the
medial head of the gastrocnemius to the popliteal surface,
• The vastus intermedjus arises from the upper three-fourths of the anterior and lateral surfaces.
• The articularis genu arises just below the vastus intermedius.
• The lower 5 cm of the anterior surface are related to suprapatellar bursa.
• The vastus lateralis arises from the upper part of the intertrochanteric line, anterior and inferior
borders of the greater trochanter, the lateral lip of the gluteal tuberosity, and the upper half of
the lateral lip of the linea aspera.
• The vastus medialis arises from the lower part of the intertrochanteric line, the spiral line, the
medial lip of the linea aspera, and the upper one-fourth of the medial supracondylar line,
• The deeper fibres of the lower half of the gluteus maximus are inserted into the gluteal
tuberosity,
• The adductor longus is inserted along the medial lip of the linea aspera between the vastus
medialis and the adductors brevis and magnus
• The adductor brevis is inserted into a line extending from the lesser trochanter to the upper
part of the linea aspera, behind the pectineus and the upper part of the adductor longus.
• The adductor magnus is inserted into the medial margin of the gluteal tuberosity, the linea
aspera, the medial supracondylar line, and the adductor tubercle, leaving a gap for the popliteal
vessels.
• The pectineus is inserted on a line extending from the lesser trochanter to the linea aspera.
• The short head of the biceps femoris arises from the lateral lip of the linea aspera between the
vastus lateralis and the adductor magnus, and from the upper two-thirds of the lateral
supracondylar line,
• The medial and lateral intermuscular septa are attached to the lips of the linea aspera and to
the supracondylar lines. They separate the extensor muscles from the adductors medially, and
from the flexors laterally.
• The lower end of the lateral supracondylar line gives origin to the plantaris above and the upper
part of the lateral head of the gastrocnemius below,
• The popliteal surface is covered with fat and forms the floor of the popliteal fossa.
• The origin of the medial head of the gastrocnemius extends to the popliteal surface just above
the medial condyle.
Lateral condyle • The fibular collateral ligament of the knee joint.
• The popliteus arises from the deep anterior part of the popliteal groove.
• The muscular impression near the lateral epicondyle gives origin to the lateral head of the
gastrocnemius.
Medial condyle • The tibial collateral ligament of the knee joint.
• The adductor tubercle receives the insertion of the hamstring part or the ischial head of the
adductor magnus.
Intercondylar notch • The anterior cruciate ligament is attached to the posterior part of the medial surface of the
lateral condyle, on a smooth impression.
• The posterior cruciate ligament is attached to the anterior part of the lateral surface of the
medial condyle, on a smooth impression.
• The intercondylar line provides attachment to the capsular ligament and laterally to the oblique
popliteal ligament.
• The infrapatellar synovial fold is attached to the anterior border of the intercondylar fossa.

144
Medico Express Block 2 GROSS ANATOMY

Ossification:
The femur ossifies from:
One primary centre: For the shaft: Appears in the seventh week of intrauterine life.
Four secondary centres: One for the lower end At the end of the ninth month of intrauterine
life

145
Medico Express Block 2 GROSS ANATOMY
One for the head During the first six months of life
One for the greater trochanter During the fourth year
One for the lesser trochanter During the twelfth year
There are:
Three epiphyses At the upper end Lesser trochanter,
Greater trochanter and
Head, in that order,
Fuse with the shaft at about eighteen years.
One epiphysis At the lower end Fuses by the twentieth year.

The following points are noteworthy:


Neck represents the upper end of the shaft because it ossifies from the primary centre.
Ossification of the lower end of the femur is of medicolegal importance.
Presence of its centre in a newly born child found dead indicates that the child was viable, i.e., it was capable of
independent existence.
The lower end of the femur is the growing end. The lower epiphyseal line passes through the adductor tubercle.
The epiphyseal line of the head coincides with the articular margins, except superiorly where a part of the non-
articular area is included in the epiphysis for passage of blood vessels to the head. In addition, the plane of this
epiphysis changes with age from an oblique to a more vertical one.

Correlation of Femoral Fractures with Blood Supply and AVN


1. Subcapital Fractures (Intracapsular Neck Fractures)
o Location: Just below the head of the femur, within the capsule of the hip joint.
o Blood Supply Compromised:
▪ These fractures often disrupt the retinacular branches of the medial femoral circumflex artery,
which ascend along the femoral neck.
▪ In adults, the epiphyseal cartilage is absent, so the femoral head is entirely dependent on these
vessels.
o Consequence:
▪ The small artery in the ligament of the head of femur (from the obturator artery) provides
inadequate collateral supply.
▪ Result: High risk of avascular necrosis (AVN) of the femoral head.
o Common In: Elderly women, especially postmenopausal due to osteoporosis.
2. Trochanteric Fractures (Extracapsular)
o Location: Through the greater and lesser trochanters.
o Blood Supply:
▪ The retinacular arteries remain intact, as the fracture is outside the capsule.
o Consequence:
▪ AVN is uncommon.
146
Medico Express Block 2 GROSS ANATOMY
▪ Healing potential is good if alignment is restored.
o Common In: Young and middle-aged adults due to direct trauma.
3. Fractures of the Shaft of Femur
o Blood Supply: These do not affect the femoral head’s vascular supply.
o Consequence: No risk of AVN to the femoral head.
o Clinical Concern:
▪ Muscle pull causes significant displacement and shortening.
▪ In distal third fractures, the gastrocnemius may compress the popliteal artery, risking vascular
compromise to the leg and foot, but not the femoral head.

147
Medico Express Block 2 GROSS ANATOMY

Sex differences:
Features: In Male: In Female:
1. Neck-shaft angle/ more Less
Angle of inclination
2. Torsion angle/ less More
Angle of declination/
Angle of anteversion
Clinical significance:
➢ COXA VARA:
• Coxa vera is a condition in which the neck-shaft angle is reduced from the normal angle of about 150° in a
child, and 127° in an adult.
• Coxa vara causes a mild shortening of the lower limb and limits passive abduction of the hip.
➢ COXA VALGA:
• Coxa valga is a condition in which the neck-shaft angle is increased.

SUMMARY
Basic Anatomy and Side Determination
• The femur is the longest and strongest bone in the body.
• Side determination: Head directed medially, shaft convex anteriorly, upper end rounded, lower end
expanded with condyles.
• It consists of the upper end (head, neck, trochanters), shaft, and lower end (condyles, intercondylar
fossa).
148
Medico Express Block 2 GROSS ANATOMY
Important Bony Features
• Upper End: Head (with fovea), neck (with ~125° neck-shaft angle), greater & lesser trochanters.
• Shaft: Cylindrical, contains linea aspera.
• Lower End: Medial/lateral condyles, patellar and tibial articular surfaces, intercondylar fossa.
• Angles: Angle of anteversion ≈ 15°; angle altered in coxa vara and coxa valga.
Muscle and Ligament Attachments
• Various muscles including gluteus medius, piriformis, iliopsoas, adductors, vastus group, gastrocnemius,
and ligaments like iliofemoral, capsular ligament, ACL/PCL are attached.
• The linea aspera is a key site for multiple muscle insertions.
Ossification and Growth
• Primary ossification for shaft at 7th intrauterine week.
• Four secondary centers: head, greater & lesser trochanters, and lower end.
• Lower end (appears before birth) is medicolegal marker of viability in neonates.
Clinical Relevance and Sex Differences
• Coxa vara: ↓ neck-shaft angle → limb shortening, restricted abduction.
• Coxa valga: ↑ neck-shaft angle.
• In females, neck-shaft angle is smaller, and anteversion is greater compared to males.
Fracture Type Location Blood Supply Affected Risk of Notes
AVN
Subcapital Neck (just below Medial femoral High Small artery in ligamentum teres
(Intracapsular) head) circumflex artery usually insufficient
Trochanteric Outside capsule Blood supply preserved Low/None Fragments well perfused
(Extracapsular)
Shaft (diaphyseal) Femoral shaft Does not affect femoral None Risk to popliteal artery in distal
head fractures

149
Medico Express Block 2 GROSS ANATOMY

REVIEW POINTS
GLUTEAL REGION
1. The gluteus maximus is the chief extensor of the hip joint.
2. It also extends and laterally rotates the hip, and helps stabilize the knee via the iliotibial tract.
3. It is an antigravity muscle innervated by the inferior gluteal nerve.
4. Gluteus medius and minimus abduct and medially rotate the thigh.
5. Gluteus medius (covered posteriorly by gluteus maximus) stabilizes the pelvis during gait.
6. Intramuscular injections in the gluteal region should be given in the upper outer quadrant to avoid sciatic nerve injury.
7. Piriformis passes solely through the greater sciatic notch and laterally rotates the thigh.
8. Other lateral rotators include obturator internus and quadratus femoris.
THIGH REGION
▪ Anterior Compartment
1. Iliopsoas (psoas major + iliacus) is the chief flexor of the hip joint.
2. Iliopsoas inserts into the anterior aspect of the lesser trochanter and laterally rotates the thigh.
3. Sartorius, rectus femoris, and vasti are innervated by the femoral nerve.
4. The femoral triangle:
o Base: inguinal ligament
o Medial boundary: medial border of adductor longus
o Lateral cutaneous nerve does not pass through it.
5. The adductor canal contains the nerve to vastus medialis and saphenous nerve.
6. The patellar (knee-jerk) reflex tests L2–L4 spinal segments via the femoral nerve.
7. Complete femoral nerve transection leads to paralysis/wasting of anterior thigh muscles.
8. The saphenous nerve, a terminal branch of femoral nerve, becomes cutaneous in the leg.
▪ Medial Compartment
9. The obturator nerve supplies the adductor muscles of the thigh.
10. It divides into anterior and posterior branches and does not extend below the knee.
11. Adductor magnus has:
o Adductor part: obturator nerve
o Hamstring part: tibial part of sciatic nerve
12. The adductor tubercle is the attachment site for the hamstring part of adductor magnus.
▪ Posterior Compartment
13. Hamstrings are innervated by the sciatic nerve and extend the hip, flex the knee.
14. Semimembranosus medially rotates thigh and leg; it also extends the hip.
15. Difficulty rising from a seated position suggests gluteus maximus paralysis (posterior thigh compensation may also be
limited).
16. Ischial tuberosity supports body weight in sitting and is the origin of hamstrings.
17. Sciatic nerve injury causes loss of knee flexion and foot movement.
KNEE JOINT
1. ACL prevents anterior displacement of tibia.
2. PCL prevents forward displacement of femur (or posterior translation of tibia).
3. The popliteus unlocks the knee at the start of flexion; its tendon is intra-articular.
4. Popliteal artery is closely related to the posterior knee capsule above the femoral condyles.
5. Supracondylar femoral fractures can injure the popliteal artery.
6. The patellar reflex tests the femoral nerve and L2–L4 roots.
7. Knee injuries are common with twisting of the femur on a fixed tibia.
LEG REGION
▪ Anterior Compartment
1. Tibialis anterior dorsiflexes the ankle and inverts the foot.
2. Peroneus tertius assists in dorsiflexion and may aid in eversion.
3. Extensor digitorum longus and hallucis longus assist in dorsiflexion and toe extension.
4. The deep peroneal nerve innervates these muscles and also supplies the first dorsal web space.
5. Injury to deep peroneal nerve causes foot drop and sensory loss between the 1st and 2nd toes.
▪ Lateral Compartment
6. Peroneus longus and brevis evert and plantarflex the foot.
7. The common peroneal nerve winds around the neck of the fibula and is vulnerable to injury.
8. Cuboid bone has a groove for the peroneus longus tendon on its plantar surface.
▪ Posterior Compartment
9. Tibialis posterior inverts the foot and supports the medial longitudinal arch.
267
Medico Express Block 2 GROSS ANATOMY

PAST MCQS
1. The medial meniscus, which is present on the inside of the knee, a) Inferior extensor retinaculum
is commonly injured due to which ligament: b) Superior extensor retinaculum
a) Anterior cruciate ligament c) Flexor retinaculum
b) Posterior cruciate ligament d) Peroneal retinaculum
c) Lateral ligament e) None
d) Medial collateral ligament Correct Answer: A
e) Fibular collateral ligament (The Y-shaped structure is indicated by the inferior extensor
Correct Answer: D retinaculum. Retinacula are bands of connective tissue that
(A medial meniscus tear is an injury to the meniscus located on the surround tendons and hold them in place. They function to
inside of the knee. It may result in pain, stiffness, swelling, stabilize tendons as the muscles to which they attach contract
locking, catching, or buckling. The medial collateral ligament is to cause movement.)
also involved in medial meniscus injury.) 7. The blood supply of hamstring muscles is provided by which of
2. What is the blood supply of the sole: the following:
a) Posterior tibial artery a) Anterior tibial
b) Femoral artery b) Posterior tibial
c) Profunda femoris artery c) Femoral
d) Anterior tibial artery d) Profunda femoris
e) Fibular artery e) Fibular artery
Correct Answer: A Correct Answer: D
(The posterior tibial artery, which is a branch of the popliteal artery, (Hamstring muscles receive vascular supply from the perforating
divides into the medial and lateral plantar arteries, which branches of the deep femoral artery, also known as the
broadly supply the sole.) profundus femoris artery.)
3. By which of the following nerves is the lateral border of the leg 8. Several muscles are used for the inversion of the foot, but which
and foot innervated: of the following is the chief invertor of the foot:
a) Lateral plantar nerve a) Peroneus Tertius
b) Sural nerve b) Peroneus Brevis
c) Saphenous nerve c) Tibialis posterior
d) Sciatic nerve d) Tibialis anterior
e) Fibular nerve e) Peroneus longus
Correct Answer: B Correct Answer: D
(The sural nerve is a peripheral nerve that arises in the posterior (The muscle used as an invertor of the foot is the tibialis anterior.)
compartment of the leg. It is formed from the combination of 9. A major part of the dorsum of the foot is supplied by which of
two smaller sensory nerves: the medial and lateral sural the following nerves:
cutaneous nerves. The medial sural cutaneous nerve is a a) Tibial nerve
terminal branch of the tibial nerve, and the lateral sural b) Deep peroneal nerve
cutaneous nerve is a terminal branch of the common peroneal c) Sciatic nerve
nerve.) d) Superficial peroneal nerve
4. If the right foot is off the ground, the right side of the pelvis is e) Femoral nerve
supported by which of the following muscles: Correct Answer: D
a) Gluteus medius of the left side (Branches of the superficial peroneal nerve supply the major
b) Gluteus maximus of the left side portion of the dorsum of the foot and toes, except the areas
c) Gluteus minimus of the left side supplied by the deep peroneal nerve and sural nerve.)
d) Gluteus minimus of the right side 10. The attachment of the obturator externus muscle takes place:
e) Gluteus medius of the right side a) Iliac fossa
Correct Answer: A b) Iliac crest
(The gluteus medius works to maintain the side of the pelvis that c) Trochanteric fossa
drops, allowing the other limb to swing forward for the next d) Greater trochanter
step. It also supports the pelvis during gait with assistance e) Lesser trochanter
from the gluteus minimus and tensor fasciae latae.) Correct Answer: C
5. A man twisted his ankle. Which ligament of the ankle joint is (It is found on the anterior aspect of the obturator foramen,
easily breakable: attached to the obturator membrane and the adjacent margin
a) Tibiotalar of the obturator foramen.)
b) Lateral ligament
c) Medial ligament 11. By which muscles is the anterior boundary of the adductor canal
d) Talofibular formed:
e) Talocalcaneal a) Adductor longus
Correct Answer: D b) Vastus medialis
(The anterior talofibular ligament is the weakest ligament of the c) Sartorius
lateral ligament complex, and approximately 70% of lateral d) Subsartorial plexus
ankle sprains involve only this ligament with a mechanism of e) Vastus lateralis
plantar flexion and inversion.) Correct Answer: B
(The anterior boundary of the adductor canal is formed by vastus
6. Which of the retinacula of the lower limb is Y-shaped: medialis.)

269
Medico Express Block 2 EMBRYOLOGY

EMBRYOLOGY
MS-A-065 Human Embryology (Development of Muscles) Learning Objectives
Name the molecular and genetic factors involved in the development of musculoskeletal system
Describe the development of skeletal muscle and innervation of axial skeleton muscles-developmental
basis of myotomes
Briefly discuss the development of cardiac and smooth muscles.

Molecular and genetic factors involved in the development of musculoskeletal system


Molecular Factors
1. Growth Factors and Signaling Pathways:
• Bone Morphogenetic Proteins (BMPs): Essential for bone and cartilage development. BMPs induce the
formation of bone and cartilage from mesenchymal stem cells.
• Transforming Growth Factor-beta (TGF-β): Regulates proliferation and differentiation of chondrocytes and
osteoblasts.
• Fibroblast Growth Factors (FGFs): Important in the development of limbs and skeletal tissues.
• Wnt Signaling Pathway: Crucial for the regulation of bone mass and remodeling. Wnt signaling promotes
osteoblast differentiation and activity.
2. Extracellular Matrix (ECM) Proteins:
• Collagen: The main structural protein in the ECM, providing tensile strength to bones and cartilage.
• Proteoglycans: Provide compressive strength and support to the ECM.
Genetic Factors
1. Transcription Factors:
• RUNX2: Master regulator of osteoblast differentiation and bone EXPRESS HIT
formation.
• SOX9: Critical for chondrogenesis and the formation of cartilage. Overactivation of Wnt signaling
• MYOD1: Involved in myogenesis, the formation of muscular tissue. in osteoblasts leads to
2. Homeobox (Hox) Genes: sclerosteosis and high bone
• Hox Genes: Regulate the patterning and formation of limbs and mass syndromes; its inhibition is
vertebrae. They provide positional information and ensure the correct a therapeutic target in
development of skeletal elements. osteoporosis.
Skeletal Muscles
1. Origin of Skeletal Muscles:
• Skeletal muscles arise from paraxial mesoderm, specifically from somites (in the occipital and trunk regions) and
somitomeres (in the head region).
• Each somite differentiates into:
o Sclerotome (forms vertebrae and ribs)
o Dermatome (forms dermis of the skin)
o Myotome (forms skeletal muscles)
2. Myotome Development:
• The myotome forms from the dorsomedial cells of the somite.
• These cells express the MYF5 gene and give rise to the epimeric (epaxial) musculature — muscles of the vertebral column
(e.g., erector spinae).
• The dorsolateral cells express the MYO-D gene and migrate to form the hypomeric (hypaxial) musculature — muscles
of the limbs and body wall.
3. Molecular Regulation:
• Signals from surrounding tissues regulate muscle gene expression:
o BMP4 and FGFs from lateral plate mesoderm
o WNT proteins from ectoderm and neural tube
• These signals activate MYO-D and MYF5, which initiate myoblast differentiation.
• Myogenin and MRF4 promote myotube and myofiber formation.
4. Myoblast Differentiation:
• Myoblasts (muscle precursor cells) fuse to form multinucleated myotubes, which mature into striated skeletal muscle
fibers.
• Cross-striations appear by the third month of development.
5. Innervation of Axial Muscles:

283
Medico Express Block 2 EMBRYOLOGY
• Each myotome is innervated by a spinal nerve that divides into:
o Dorsal primary ramus → innervates epaxial muscles (derived from epimere)
o Ventral primary ramus → innervates hypaxial muscles (derived from hypomere)
• This segmental innervation persists even as muscles migrate during development.
6. Muscle Patterning:
• The connective tissue (derived from:
o Somitic mesoderm in trunk,
o Neural crest in the head,
o Somatic mesoderm in limbs)
directs the regional pattern of muscle formation.
7. Final Derivatives of Myotomes:
Term Description
Epaxial Muscles derived from the epimere (dorsomedial myotome); become the deep back (extensor) muscles of
muscles the vertebral column.
Hypaxial Muscles derived from the hypomere (ventrolateral myotome); form muscles of limbs, thorax, abdomen,
muscles neck, etc.
Primaxial Muscle precursors that remain near the neural tube; give rise to epaxial + some hypaxial muscles that do
domain not migrate across the lateral somitic frontier.
Abaxial Muscle precursors that migrate across the lateral somitic frontier into lateral plate mesoderm; form most
domain hypaxial muscles (especially in limbs and body wall).
During embryonic development, skeletal muscles develop from the paraxial mesoderm, which forms somites. Each somite
differentiates into:
• Dermatome (forms dermis)
• Myotome (forms skeletal muscles)
• Sclerotome (forms vertebrae)
Primaxial Muscles Abaxial Muscles
Deep back muscles (e.g., erector spinae) Limb muscles (upper & lower limbs)
Intercostal muscles Abdominal wall muscles (external/internal oblique, rectus abdominis)
Scalene muscles Infrahyoid muscles
Prevertebral muscles Muscular part of diaphragm
Crural part of diaphragm
Tongue muscles (from occipital myotomes)
• Epimere (dorsomedial part) → extensor muscles of the spine (epaxial muscles).
• Hypomere (dorsolateral part) → muscles of:
o Neck: scalene, geniohyoid, prevertebral
o Thorax: intercostal muscles
o Abdomen: external & internal obliques, transversus abdominis
o Pelvis: pelvic diaphragm, anal muscles
o Limbs: limb musculature (via migration into limb buds)
PAST SEQ: a) In the development of skeletal muscles what are the Primaxial andAbaxial domains?
b) In a tabulated form write the origins of muscles of various regions from abaxial and Primaxial precursors. (Annual 2023 held
in 2024)
Cardiac Muscle Development:
• Origin: Cardiac muscle develops from splanchnic mesoderm surrounding the endothelial heart tube.
• Myoblast Characteristics:
o Do not fuse (unlike skeletal muscle).
o Develop intercalated discs from special cell junctions between adjacent myoblasts.
o Myofibrils form in a manner similar to skeletal muscle.
• Conducting System:
o Some cardiac myocytes differentiate into Purkinje fibers.
o These cells have fewer myofibrils and form the conducting system of the heart.
Smooth Muscle Development:
• Gut and Its Derivatives:
o Derived from splanchnic mesoderm surrounding the endoderm of the gut tube.
• Vascular Smooth Muscle:
o Arises from mesoderm located adjacent to vascular endothelium.
• Other Smooth Muscles:
o Sphincter and dilator muscles of the pupil, as well as muscles of mammary and sweat glands, originate from
ectoderm.

284
Medico Express Block 2 EMBRYOLOGY
SUMMARY
Molecular and Genetic Factors:
Molecular Signals:
• BMPs, TGF-β, FGFs: Regulate bone, cartilage, and muscle development.
• Wnt signaling: Promotes osteoblast formation and bone remodeling.
• ECM proteins:
o Collagen: Provides tensile strength.
o Proteoglycans: Provide compressive support.
Key Transcription Factors:
• RUNX2: Essential for osteoblast differentiation.
• SOX9: Required for cartilage formation.
• MYOD1, MYF5: Control skeletal muscle formation.
Hox Genes:
• Direct patterning of limbs and vertebrae, giving positional identity.
Skeletal Muscle Development:
• Originates from paraxial mesoderm (somites and somitomeres).
• Somites form:
o Sclerotome → vertebrae & ribs
o Dermatome → dermis
o Myotome → skeletal muscle
Myotome Derivatives:
• Epimere (dorsomedial) → epaxial (spinal extensor) muscles
• Hypomere (dorsolateral) → hypaxial muscles (limbs, body wall, etc.)
Regulation:
• WNT, BMP4, FGFs activate MYOD, MYF5, leading to myoblast differentiation.
• Myogenin & MRF4: Promote formation of myotubes and myofibers.
Innervation:
• Dorsal rami → epaxial muscles
• Ventral rami → hypaxial muscles
• Nerves remain with migrating muscles.
Muscle Patterning:
• Controlled by connective tissue:
o Neural crest (head), somitic mesoderm (trunk), somatic mesoderm (limbs)
Cardiac Muscle Development:
• Origin: Splanchnic mesoderm around the heart tube.
• Features:
o Myoblasts do not fuse.
o Develop intercalated discs.
o Form Purkinje fibers for cardiac conduction.
Smooth Muscle Development:
• Gut wall: From splanchnic mesoderm around the gut.
• Blood vessels: From adjacent mesoderm.
• Pupil, sweat & mammary glands: From ectoderm.

285
Medico Express Block 2 EMBRYOLOGY

MS-A-066 Human Embryology (Development of Limb) Learning Objectives


Describe the process of limb development and limb growth
Limb Development and Growth
1. Initiation of Limb Development
• Begins at the end of the 4th week of embryonic life.
• Limb buds appear as outpouchings from the ventrolateral body wall.
• Each bud consists of:
o Mesenchymal core (from somatic lateral plate mesoderm): forms bones and connective tissue.
o Cuboidal ectoderm covering.
2. Apical Ectodermal Ridge (AER) and Progress Zone
• AER forms at the distal tip of the limb bud.
• AER induces adjacent mesenchyme to remain proliferative (progress zone).
• Proximodistal growth occurs as cells further from AER begin to differentiate into cartilage and muscle.
3. Segmentation and Digit Formation
• 6th week: Distal end flattens into handplates and footplates.
• Constrictions appear, dividing limb into segments (arm/forearm, thigh/leg).
• Digit formation:
o AER separates into five segments due to cell death.
o Mesenchyme condenses into digital rays.
o Apoptosis between rays separates individual fingers and toes.
4. Limb Rotation
• Occurs during the 7th week:
o Upper limb: rotates 90° laterally – extensors move posterior/lateral, thumb is lateral.
o Lower limb: rotates 90° medially – extensors face anteriorly, big toe is medial.
EXPRESS HIT
5. Cartilage and Bone Formation
• Mesenchyme differentiates into chondrocytes → form hyaline cartilage models Persistent lateral rotation of
(6th week). upper limbs may explain
• Endochondral ossification begins: postural anomalies like 'waiter's
o Primary ossification centers appear in the diaphysis (by 12th week). tip' in brachial plexus injury
o Secondary ossification centers appear in the epiphysis (after birth). mimics.
• Epiphyseal plate (growth plate) remains temporarily between diaphysis and
epiphysis:
o Responsible for longitudinal bone growth.
o Disappears once bone reaches full length.
6. Joint Formation
• Occurs at regions of arrested chondrogenesis → forms joint interzones.
• Joint cavity forms via cell death.
• Surrounding mesenchyme → joint capsule.
• WNT14 is involved in joint formation signaling.

286
Medico Express Block 2 HISTOLOGY

HISTOLOGY
MS-A-070 Histology/Integrate with Medicine (Histology of Muscles) Learning Objectives
Describe the microscopic structure and ultramicroscopic structure of skeletal muscle
Explain the basis of myasthenia gravis.
Describe the microscopic and ultramicroscopic structure of cardiac muscle
Describe the microscopic and ultramicroscopic structure of smooth muscle
Compare and contrast the histological features of three types of muscle tissue

Skeletal Muscle
Microscopic Structure:
1. Muscle Fibers:
• Long, cylindrical cells.
• Multinucleated with nuclei located peripherally beneath the sarcolemma.
• Diameters range from 10-100 μm.
2. Connective Tissue Layers:
• Epimysium: Dense irregular connective tissue surrounding the entire muscle.
• Perimysium: Thin connective tissue surrounding bundles of muscle fibers (fascicles).
• Endomysium: Delicate layer of reticular fibers and fibroblasts surrounding individual muscle fibers within
fascicles.
3. Myofibrils and Sarcomeres:
• Myofibrils: Long cylindrical filament bundles within muscle fibers. EXPRESS HIT
• Sarcomeres: Functional units of myofibrils, extending from Z disc to Z Epimysial fibrosis disrupts
disc, about 2.5 μm long.
global force transmission in
• A bands (dark bands) and I bands (light bands) form striations.
chronic myopathies like
4. Thick and Thin Filaments:
Duchenne, impairing entire
• Thick filaments (myosin) occupy A band.
muscle unit contractility.
• Thin filaments (actin) span I band.
• Arranged in a pattern giving rise to striations.

Ultramicroscopic Structure:
1. Myosin and Actin Filaments:
• Myosin filaments: 1.6 μm long, 15 nm wide, composed of myosin molecules arranged with overlapping rodlike
portions.
• Actin filaments: 1.0 μm long, 8 nm wide, containing G-actin monomers with binding sites for myosin.
2. Sarcomere Components:
299
Medico Express Block 2 HISTOLOGY
• Z disc: Bisects I bands, serve as attachment sites for actin filaments.
• M line: Bisects H zone, contains myomesin and creatine kinase.
• H zone: Region within A band containing only myosin filaments.
• Titin: Large accessory protein providing structural support and elasticity, connecting thick filaments to Z disc.

3. Regulatory Proteins:
• Tropomyosin: Coil of polypeptide chains located in the EXPRESS HIT
groove between actin strands.
• Troponin: Complex of three subunits (TnT, TnC, TnI) Myofibrillar disarray, especially near Z discs,
regulating actin-myosin interaction and binding calcium is diagnostic in desmin-related
ions. myopathies—causing progressive weakness
and cardiomyopathy.

300
Medico Express Block 2 HISTOLOGY

4. Sarcoplasmic Reticulum and Transverse Tubule System:


• Sarcoplasmic reticulum: Surrounds myofibrils, and contains pumps for calcium sequestration.
• Transverse tubules (T-tubules): Invaginations of the sarcolemma, form triads with terminal cisternae of the
sarcoplasmic reticulum, facilitating calcium release for muscle contraction.
PAST SEQ: What is T-system in skeletal muscle? What are its two important functions? 1, 1 (Supplementary 2017 held in 2018)

301
Medico Express Block 2 HISTOLOGY
Myasthenia Gravis
• Myasthenia gravis is an autoimmune disorder involving circulating antibodies against acetylcholine receptor proteins.
Mechanism:

Antibody binding to the antigenic sites interferes with acetylcholine activation

As the body attempts to correct the condition, junctional folds of sarcolemma with affected receptors are
internalized, digested by lysosomes, and replaced by newly formed receptors.

These receptors are again made unresponsive to acetylcholine by similar antibodies, and the disease
follows a progressive course.

Symptoms of Myasthenia Gravis:


1. Muscle Weakness and Fatigue EXPRESS HIT
2. Ptosis (Drooping Eyelids)
3. Diplopia (Double Vision) Fluctuating fatigability—not
4. Difficulty Swallowing (Dysphagia) constant weakness—is
5. Speech Difficulties pathognomonic for MG and
6. Facial Weakness worsens with repetitive use
7. Muscle Weakness in Limbs due to receptor depletion.
8. Respiratory Muscle Weakness
9. Worsening of Symptoms with Activity
Integration with Medicine for Myasthenia Gravis:
1. Medication Management
• Acetylcholinesterase Inhibitors: Enhance neuromuscular transmission.
• Immunosuppressants: Suppress the immune system.
• Monoclonal Antibodies: Target antibody-producing cells.
2. Thymectomy
• Surgical removal of the thymus gland to improve symptoms. EXPRESS HIT
3. Plasma Exchange (Plasmapheresis)
• Removes circulating antibodies temporarily. Thymic hyperplasia or
4. Intravenous Immunoglobulin (IVIg) Therapy thymoma is present in over
• Provides antibodies from healthy donors to modulate the immune 70% of MG patients, linking
response. MG to paraneoplastic
5. Respiratory Support autoimmunity.
• Mechanical ventilation for severe respiratory muscle weakness.
6. Symptomatic Treatment
• Eyelid crutches or prisms for ptosis and double vision correction.
Cardiac muscle
Microscopic Structure:
• Complex junctions between interdigitating processes.
• Branch and join with adjacent fibers, forming tightly knit bundles.
• Cell diameter: 15-30 μm.
• Cell length: 85-120 μm.
• One centrally located nucleus per cell.
• Exhibits a striated banding pattern
• Surrounding muscle cells is a delicate sheath of endomysium with a rich capillary network.
• Thicker perimysium separates bundles and layers of muscle fibers.

302
Medico Express Block 2 HISTOLOGY

Ultramicroscopic Structure:
1. Intercalated Discs:
• Transverse lines at irregular intervals where myocardial cells join.
• Consists of many desmosomes and fascia adherens junctions for
strong intercellular adhesion. EXPRESS HIT
• Longitudinally oriented regions contain gap junctions for electrical
Mitochondria occupy up to
synapses.
40% of cytoplasmic volume
2. Contractile Apparatus:
in ventricular cells,
• Similar to skeletal muscle, containing myofibrils with sarcomeres.
underscoring resistance to
• Mitochondria occupy up to 40% of the cell volume.
• Presence of triglyceride droplets, glycogen granules, and lipofuscin fatigue.
pigment granules.
3. T-tubules and Sarcoplasmic Reticulum:
• T-tubules are well-developed in ventricular muscle fibers, less so in atrial muscle.
• The sarcoplasmic reticulum is less organized compared to skeletal muscle fibers, forming dyads instead of
triads in TEM sections.
4. Contraction Mechanism:
• Intrinsic and spontaneous contraction regulated by specialized myocardial fibers.
• Autonomic innervation modulates the rate of contraction.

303
Medico Express Block 2 HISTOLOGY
Smooth muscle
Microscopic Structure:
• Complex junctions between interdigitating processes.
• Cells often branch and join with adjacent fibers, forming tightly knit bundles.
• Diameter: 15-30 μm.
• Length: 85-120 μm.
• one centrally located nucleus per cell.
• No striated banding pattern.
• Enclosed by endomysium containing type I and type III collagen fibers.

Ultramicroscopic Structure:
1. Sarcoplasmic Reticulum and T-tubules: EXPRESS HIT
• Rudimentary sarcoplasmic reticulum, lacks T-tubules.
2. Cellular Organelles: Slow, sustained contractions
• Mitochondria, polyribosomes, RER, and Golgi apparatus concentrated are energy-efficient due to
near the nucleus. the "latch state" of myosin-
• Short plasmalemma invaginations resembling caveolae. actin binding.
3. Contractile Mechanism:
• Actin and myosin are organized differently from striated muscle.
• Bundles of thin and thick myofilaments crisscross the sarcoplasm obliquely.
• Uses calmodulin and Ca2+-sensitive myosin light-chain kinase for contraction.
4. Intermediate Filaments:
• 10-nm intermediate filaments composed of desmin.
5. Cellular Junctions:
• Dense bodies containing α-actinin functionally similar to Z discs in striated muscle.
• Cadherins of desmosomes linking adjacent smooth muscle cells.
6. Neuromuscular Junctions:
• Lack of well-defined neuromuscular junctions.
• Contraction stimulated by autonomic nerves or paracrine secretions.
• Axons of autonomic nerves have periodic swellings (varicosities) that release neurotransmitters.

304
Medico Express Block 2 HISTOLOGY

Important comparisons of the three types of muscle.


Characteristic Skeletal Muscle Cardiac Muscle Smooth Muscle
Fibers Single multinucleated cells Aligned cells in branching Single small, closely packed fusiform
arrangement cells
Cell/fiber shape Cylindrical, 10-100 μm Cylindrical, 10-20 μm diameter, Fusiform, diameter 0.2-10 μm,
and size diameter, many cm long 50-100 μm long length 50-200 μm
Striations Present Present Absent
Location of Peripheral, adjacent to the Central Central, at the widest part of the
nuclei sarcolemma cell
T tubules Center of triads at A-I junctions In dyads at Z discs Absent; caveolae may be
functionally similar
Sarcoplasmic Well-developed, with two Less well-developed, one small Irregular smooth ER without
Reticulum (SR) terminal cisterns per terminal cistern per sarcomere distinctive organization
sarcomere in triads with T in a dyad with T tubule
tubule
Special Very well-organized Intercalated discs joining cells, Gap junctions, caveolae, dense
structural sarcomeres, SR, and transverse with many adherent and gap bodies
features tubule system junctions
Control of Troponin C binds Ca2+, moving Similar to that of skeletal muscle Actin-myosin binding occurs with
contraction tropomyosin and exposing myosin phosphorylation by MLCK
actin for myosin binding triggered when calmodulin binds
Ca2+
Connective Endomysium, perimysium, and Endomysium; subendocardial Endomysium and less-organized CT
tissue epimysium and subpericardial CT layers sheaths
organization
Major locations Skeletal muscles, tongue, Heart Blood vessels, digestive and
diaphragm, eyes, and upper respiratory tracts, uterus, bladder,
esophagus and other organs
Key function Voluntary movements Automatic (involuntary) Involuntary movements
pumping of blood
Efferent Motor Autonomic Autonomic
innervation
Contractions All-or-none triggered at motor All-or-none, intrinsic (beginning Partial, slow, often spontaneous,
end plates at nodes of conducting fibers) wavelike, and rhythmic
Cell response to Hypertrophy (increase in fiber Hypertrophy Hypertrophy and hyperplasia
increased load size) (increase in cell/fiber number)
Capacity for Limited, involving satellite cells Very poor Good, involving the mitotic activity
regeneration mainly of muscle cells

305
Medico Express Block 2 HISTOLOGY
Diagrams

PAST SEQ: Q1. a) Compare the histological features of skeletal and smooth muscles. 3
Give histological difference between cardiac and skeletal muscles (Annual 2024)

SUMMARY
Skeletal Muscle
• Fibers: Long, cylindrical, multinucleated (peripheral nuclei), striated.
• Connective tissue:
o Epimysium (entire muscle),
o Perimysium (fascicles),
o Endomysium (individual fibers).
• Myofibrils: Composed of sarcomeres (Z disc to Z disc, ~2.5 μm).
• Filaments:
o Thick: Myosin (A band),
o Thin: Actin (I band).
• SR & T-tubules: Form triads, control Ca²⁺ for contraction.
• Regulatory proteins: Troponin (TnT, TnC, TnI), Tropomyosin.
Myasthenia Gravis
• Autoimmune disease: Antibodies block acetylcholine receptors.
• Symptoms: Ptosis, diplopia, dysphagia, limb/respiratory weakness.
• Treatment:
o Acetylcholinesterase inhibitors,
o Immunosuppressants,
o Monoclonal antibodies,
o Plasmapheresis, IVIg, Thymectomy.
Cardiac Muscle
• Striated, branched fibers with central nucleus.
• Intercalated discs: Contain desmosomes, gap junctions (for electrical conduction).
• T-tubules form dyads with SR (less organized than skeletal).
• High mitochondria content (~40% cell volume).
• Spontaneous contraction, controlled by autonomic nerves.
Smooth Muscle
• Non-striated, spindle-shaped cells, central nucleus.
• No T-tubules; caveolae functionally similar.
• Contraction via calmodulin + MLCK (not troponin).
• Dense bodies (anchor filaments), gap junctions for coordination.
• Innervated by autonomic nerves, with neurotransmitters released from varicosities.

306
Medico Express Block 2 PHYSIOLOGY

PHYSIOLOGY
MS-P-001 Medical Physiology (Diffusion / Equilibrium Potentials & Nernst potential) Learning Objectives
Explain the Physiological basis of membrane potential
Explain diffusion potentials of Na & K
BASIC PHYSICS OF MEMBRANE POTENTIALS
MEMBRANE POTENTIALS CAUSED BY ION CONCENTRATION
Potassium • The potassium concentration is great inside a nerve fibre membrane but very low outside the membrane
ions that builds up a concentration gradient.
• This gradient causes potassium ions to move outward, thus creating electropositivity outside the membrane
and electronegativity inside because of negative anions that remain behind and do not diffuse outward with
the potassium.
• Within a millisecond or so, the potential difference between the inside and outside, called the diffusion
potential, becomes great enough to block further net potassium diffusion to the exterior
• In the normal mammalian nerve fibre, the potential difference is about 94 millivolts, with negativity inside
the fibre membrane.
Sodium ions • High concentration of Sodium ions outside the membrane and a low concentration of sodium ions inside.
• Diffusion of the positively charged sodium ions to the inside creates a membrane potential of opposite
polarity with negativity outside and positivity inside.
• Again, the membrane potential rises high enough within milliseconds to block further net diffusion of
sodium ions to the inside.
• however, this time, in the mammalian nerve fibre, the potential is about 61 millivolts positive inside the
fibre

A: Establishment of a diffusion potential across a nerve fibre membrane,


caused by diffusion of potassium ions from inside the cell to outside through a
membrane that is selectively permeable only to potassium.
B: Establishment of a diffusion potential when the nerve fibre membrane is
permeable only to sodium ions. Note that the internal membrane potential is
negative when potassium ions diffuse and positive when sodium ions diffuse
because of opposite concentration gradients of these two ions.

SUMMARY
• Potassium-driven Membrane Potential: A high intracellular potassium concentration and selective permeability
to potassium cause K⁺ to diffuse outward, leaving behind negatively charged anions. This creates a negative
membrane potential (~–94 mV) inside the cell due to electrochemical equilibrium.
• Sodium-driven Membrane Potential: When the membrane is selectively permeable to sodium, Na⁺ diffuses
inward from its higher extracellular concentration, making the inside of the cell positively charged (~+61 mV)
until the diffusion potential blocks further sodium movement.

325
Medico Express Block 2 PHYSIOLOGY

MS-P-002 Medical Physiology (Nernst potential) Learning Objectives


Define Nernst potential
Explain Physiological Basis of Nernst potential
Write the Nernst equation
Calculate Nernst potential for Na & K
Explain the effects of altering the concentration of Na+, K+, Ca on the equilibrium potential for that ion

NERNST POTENTIAL
Definition The diffusion potential level across a membrane that exactly opposes the net diffusion of a particular
ion through the membrane is called the Nernst potential.
Physiological basis The magnitude of the Nernst potential is determined by the ratio of the concentrations of that
specific ion on the two sides of the membrane.
The greater this ratio, the greater the tendency for the ion to diffuse in one direction, and therefore
the greater the Nernst potential required to prevent additional net diffusion
formula

Nernst potential for K outside 4 mEq/L


potassium K inside 140 mEq/L
K (inside) /K (outside) = 35.0
The logarithm of 35 is 1.54, and this multiplied by −61 millivolts is −94 millivolts.
Nernst potential for Na outside 142mEq/L
sodium Na inside 14mEq/L
Na (inside)/Na (outside) = 0.1
The logarithm of 0.1 is 0.1.
Nernst potential for the inside of the membrane of +61 millivolts
PAST SEQ: Define NERNST Potential. What is the role of potassium and Na-K ATPase pump.

Equilibrium potential
The potential at which there is no net movement of that ion across the membrane. If the concentrations on the two sides are
equal, the force of the concentration gradient would be zero, and the equilibrium potential would also be zero. The larger the
concentration gradient, the larger the equilibrium potential.
Effect of altering the concentration of sodium, potassium and calcium.
Sodium Since Na+ is high outside the neuron, the concentration gradient favors the entry of Na+ into the cell.
The equilibrium potential for Na+ is about 60 mV, as a positive internal charge would oppose the entry
of Na+
when Na+ channels open and then the equilibrium potential for sodium is changed namely less
positive.
Potassium For potassium, as the equilibrium potential becomes more positive, the resting potential becomes
more positive (i.e., more depolarized).
Calcium Calcium ions contribute little to the resting membrane potential in most cells despite the fact that
they have a large transmembrane concentration gradient and hence a very positive
equilibrium potential.

SUMMARY
Nernst Potential & Equilibrium Concept: The Nernst potential is the membrane voltage that exactly balances an
ion’s concentration gradient, preventing its net diffusion. It depends on the concentration ratio of the ion across the
membrane and is calculated using the Nernst equation. For example, K⁺ has a Nernst potential of –94 mV, while Na⁺
has +61 mV based on typical intracellular and extracellular concentrations.
Effects of Ion Concentrations on Membrane Potential:
• Sodium (Na⁺): High outside; tends to enter the cell, driving the potential toward +60 mV.
• Potassium (K⁺): High inside; tends to leave the cell, making the membrane more negative (hyperpolarized).
• Calcium (Ca²⁺): Although it has a steep gradient, its contribution to resting potential is minimal due to low
permeability.

326
Medico Express Block 2 PHYSIOLOGY

MS-P-003 Medical Physiology (Goldman Equation) Learning Objectives


Describe the normal distribution of Na+, K+, Ca and Cl- across the cell membrane
Explain physiological basis of Goldman equation
Clarify the role of Goldman equation in generation of Resting Membrane Potential (RMP).

Normal Distribution of Ions Across the Cell Membrane:


1. Sodium (Na⁺):
o High outside the cell (~145 mEq/L)
o Low inside the cell (~10–15 mEq/L)
o Tends to move into the cell down its concentration gradient.
2. Potassium (K⁺):
o High inside the cell (~140 mEq/L)
o Low outside the cell (~4–5 mEq/L)
o Tends to move out of the cell down its concentration gradient.
3. Calcium (Ca²⁺):
o Very high outside the cell (~1.2 mM)
o Extremely low inside the cell (~0.0001 mM or 100 nM)
o Tends to move into the cell; steep gradient maintained by active transport.
4. Chloride (Cl⁻):
o High outside the cell (~110 mEq/L)
o Low inside the cell (~4–30 mEq/L, varies by cell type)
o Tends to move into the cell, but its distribution is influenced by the membrane potential and other anions
inside the cell.
Physiological Basis of the Goldman Equation:
1. Multi-Ion Contribution to Membrane Potential:
The Goldman equation (also known as the Goldman-Hodgkin-Katz equation) calculates the membrane potential when
the membrane is permeable to more than one ion, typically Na⁺, K⁺, and Cl⁻.
2. Key Factors in the Equation:

It considers:
o Polarity of each ion’s charge (positive or negative),
o Concentration gradient of the ion across the membrane (inside vs. outside),
o Membrane permeability to each ion (P).
These determine the net diffusion potential across the membrane.
3. Effect of Ion Permeability:
The greater the membrane's permeability to an ion, the greater its influence on membrane potential. If only one ion is
permeable, the membrane potential equals its Nernst potential.
4. Ion Movement and Charge Effect:
o A positive ion (like K⁺) moving outward increases negativity inside the membrane.
o A negative ion (like Cl⁻) moving inward also increases negativity inside.
Thus, the direction and nature of ion movement influence the membrane charge.

Role of the Goldman Equation in Generation of Resting Membrane Potential (RMP):


1. Resting Potential Reflects Ion Gradients and Permeabilities:
At rest, the membrane is most permeable to K⁺, less to Na⁺, and moderately to Cl⁻.
The Goldman equation accounts for these differences and calculates a realistic RMP (~ –70 mV in neurons).
2. Dominance of Potassium:
Since K⁺ permeability is highest, its Nernst potential (~ –94 mV) dominates the RMP. However, small contributions
from Na⁺ (+61 mV) and Cl⁻ adjust the final RMP value.
3. Dynamic Nature of RMP:
Although called "resting", the RMP is a dynamic balance influenced by ion channel activity, transporter function, and
electrochemical gradients.
4. Electrochemical Driving Force:
The difference between the membrane potential (Vm) and each ion’s equilibrium potential (Veq) determines the
direction and magnitude of ion flow (Vdf = Vm – Veq).
This concept explains how changes in permeability (e.g. opening Na⁺ or K⁺ channels) can initiate action potentials or
regulate cellular activity.
327
Medico Express Block 2 PHYSIOLOGY
SUMMARY
Normal Ion Distribution Across Cell Membrane:
• Na⁺ and Cl⁻ are high outside the cell; K⁺ is high inside; Ca²⁺ has a very steep gradient (much higher outside).
• These gradients drive ions passively across the membrane, with Na⁺ and Ca²⁺ tending to enter, and K⁺
tending to exit.
Goldman Equation – Physiological Basis:
• It calculates membrane potential when multiple ions (Na⁺, K⁺, Cl⁻) are involved, factoring in each ion’s
charge, concentration gradient, and membrane permeability.
• The greater the permeability, the more influence an ion has on membrane potential.
Role in Resting Membrane Potential (RMP):
• RMP (~–70 mV) mainly reflects K⁺ dominance, adjusted by Na⁺ and Cl⁻.
• The equation explains how ion fluxes and channel dynamics determine membrane voltage and underlie
processes like action potential initiation.

328
Medico Express Block 2 PHYSIOLOGY

MS-P-004 Medical Physiology integrate with Anesthesiology (Resting Membrane Potential in Neurons) Learning
Objectives
Describe the Physiological basis of generation of RMP
Explain the effects of hyperkalaemia and Hypokalaemia on the Resting Membrane Potential (RMP)
Name the membrane stabilizers
Explain the physiological basis of action of Local Aesthetics

Physiological Basis of RMP:


The Resting Membrane Potential (RMP) is the electrical potential difference across the cell membrane of a resting neuron,
typically around –70 to –90 mV, with the inside of the cell more negative than the outside. This is primarily due to the unequal
distribution of ions across the membrane and their differential permeability.
Key contributors to RMP:
1. Na⁺/K⁺ ATPase Pump (Electrogenic Pump):
o Continuously pumps 3 Na⁺ out and 2 K⁺ in per cycle.
o Contributes to negative charge inside the cell.
o Maintains high [Na⁺] outside and high [K⁺] inside the cell.
o Generates –4 mV directly due to net loss of positive charges.

2. Potassium (K⁺) Diffusion (Main Contributor):


o Membrane is highly permeable to K⁺ due to leak channels.
o K⁺ tends to diffuse out of the cell (down its concentration gradient).
o Results in a negative charge inside, generating a potential of about
–94 mV (Nernst potential for K⁺).
3. Sodium (Na⁺) Diffusion (Minor Contribution):
o Membrane is slightly permeable to Na⁺ (about 1/100 as permeable
as to K⁺).
o Na⁺ tends to leak in, reducing the negativity.
o Contributes a potential of +61 mV (Nernst potential for Na⁺).
o Due to lower permeability, Na⁺ diffusion has minimal effect, resulting in a net RMP of –86 mV when combined
with K⁺ diffusion.
4. Chloride and Other Ions:
o Cl⁻ ions also contribute to the RMP depending on their gradient and passive distribution but are not the primary
determinant in neurons.
PAST SEQ: Q1: How is the resting membrane potential generated? (Annual 2007)
Q2: Outline the mechanism of resting membrane potential. What is the effect of changes extracellular fluid potassium
concentration on this potential? (Supple 2015)
Q3: Considering a nerve fiber, give the physiological basis of: (Supple 2018 held in 2019)
(a) Resting Membrane Potential (RMP).
Q4: a) What is the mechanism of the Resting Membrane Potential? What is the effect of changes in extracellular fluid potassium
concentration on this potential? (Annual 2021 held in 2022)
Q5: Explain the physiological basis of generation of resting membrane potential of a nerve fiber.
Q6: (A) Define resting membrane potential. (Annual 2013)
(B) Enumerate the three factors that explain RMP.
(C) Explain the factor that contributes the most. (permeability of the membrane to K+)

329
Medico Express Block 2 PHYSIOLOGY

Effects of Hyperkalemia and Hypokalemia on RMP:


Condition Definition Effect on RMP Physiological Consequence
Hyperkalemia Elevated extracellular RMP becomes less negative Neurons become more excitable, risk of
K⁺ (depolarized) spontaneous firing
Hypokalemia Decreased RMP becomes more negative Neurons become less excitable, harder to
extracellular K⁺ (hyperpolarized) reach threshold
Explanation:
• RMP is largely set by K⁺ gradient.
• In hyperkalemia, the gradient between intracellular and extracellular K⁺ is reduced, so less K⁺ diffuses out, and RMP
becomes less negative.
• In hypokalemia, the gradient is increased, causing more K⁺ to leave, making the inside more negative.
In Summary:
• RMP (~–70 to –90 mV) is established mainly by K⁺ efflux, slightly counteracted by Na⁺ influx, and maintained by the
Na⁺/K⁺ ATPase pump.
• Hyperkalemia leads to depolarization (↑ excitability), while hypokalemia leads to hyperpolarization (↓ excitability) of
neurons.

Membrane-stabilizing factors
• They decrease excitability.
• Calcium ions are said to be a “stabilizer”
• A high extracellular fluid calcium ion concentration decreases membrane permeability to sodium ions and simultaneously
reduces excitability.
Local Anaesthetics.
• Stabilizers
• Procaine and tetracaine.
• Act directly on the activation gates of the sodium channels, making it much more difficult for these gates to open, thereby
reducing membrane excitability.
• When excitability has been reduced so low that the ratio of action potential strength to excitability threshold (called the
“safety factor”) is reduced below 1.0, nerve impulses fail to pass along the anesthetized nerves.
SUMMARY
Generation of Resting Membrane Potential (RMP):
• RMP in large nerve fibers is about –90 mV.
• It results mainly from:
o K⁺ diffusion (major contributor, ~–94 mV),
o Na⁺ diffusion (minor, +61 mV),
o Na⁺/K⁺ ATPase pump (electrogenic effect of –4 mV).
• The Goldman equation combines Na⁺ and K⁺ diffusion potentials (~–86 mV), and with pump action, total
RMP reaches –90 mV.
Influence of Potassium and Sodium:
• The membrane is 100x more permeable to K⁺ than Na⁺, so K⁺ efflux dominates RMP.
• Hyperkalemia makes RMP less negative (hypopolarization → hyperexcitability);
Hypokalemia makes RMP more negative (hyperpolarization → hypoexcitability).
Membrane-Stabilizing Factors:
• High extracellular Ca²⁺ stabilizes the membrane by reducing Na⁺ permeability.
• Local anesthetics (e.g., procaine, tetracaine) block Na⁺ channel activation gates, lowering excitability.
• When excitability drops below threshold, action potentials fail to propagate, causing local anesthesia.

330
Medico Express Block 2 PHYSIOLOGY

MS-P-005 Medical Physiology (Neurons) Learning Objectives


Describe the Physiological anatomy of Neurons
Discuss the axonal transport
Enlist & give functions of Neuroglial cells
Explain process of myelination in Central Nervous System (CNS) & Peripheral Nervous System (PNS)

STRUCTURE OF NEURON
Neuron is made up of three parts:
1. Nerve cell • Also known as soma or perikaryon.
body • It is irregular in shape.
• Have cytoplasm called neuroplasm that contains:
1.Nucleus:
Uni-nucleated, centrally placed, two prominent nucleoli but does not contain centrosome.
(So, the nerve cell cannot multiply like other cells)
2.Nissl bodies:
small basophilic granules found in cytoplasm of neurons
These bodies are present in soma and dendrite but not in axon and axon hillock.
Contain ribosomes. So, these bodies are concerned with synthesis of proteins.
3.Neurofibrils: thread-like structures present in the form of network in the soma and the
nerve processes consist of microfilaments and microtubules.
4.Mitochondria are present in soma and in axon
5.Golgi apparatus. Protein packaging
• Nissl bodies and neurofibrils are found only in nerve cell and not in other cells
2. Dendrite • short processes
• branched process of neuron
• May be present or absent.
• Has Nissl granules and neurofibrils.
• Transmits impulses towards the nerve cell body.
3. Axon • Longer process of nerve cell.
• Each neuron has only one axon.
• Arises from axon hillock of the nerve cell body and it is devoid of Nissl granules.
• Axon extends for a long distance away from the nerve cell body.
• Length of longest axon is about 1 meter.
• Axon transmits impulses away from the nerve cell body

331
Medico Express Block 2 PHYSIOLOGY
Axonal transport
The cytoskeleton especially the microtubules play a major role in axonal transport, two proteins Tubulin and Dynein provide major
track for transportation.
Orthograde • Transport from cell body to nerve endings
• occurs along microtubules
• Requires two molecular motors, dynein and kinesin.
• fast axonal transport occurs at about 400 mm/day
• Slow axonal transport occurs at 0.5–10 mm/day
Example; transport of synaptic vesicles to the axon terminal
Retrograde • transport from the nerve ending to the cell body
• occurs along microtubules at about 200 mm/day
• Synaptic vesicles recycle in the membrane, but some used vesicles are carried back to the cell body and
deposited in lysosomes.
• Some materials taken up at the ending by endocytosis, including nerve growth factor (NGF) and some
viruses are also transported back to the cell body
Example; transport of used synaptic vesicles to the cell body gives information about the axon terminal

Formation of Myelin Sheath


Myelinogenesis: Formation of myelin sheath around the axon, it is formed by Schwann cells in neurilemma.
In Peripheral • myelinogenesis starts at 4th month of intrauterine life It is completed only in the second
Nervous System year after birth.
• Schwann cells wrap up and rotate around the axis cylinder in many concentric layers.
• The concentric layers fuse to produce myelin sheath but cytoplasm of the cells is not
deposited.
• Outermost membrane of Schwann cell remains as neurilemma.
• Nucleus of these cells remains in between myelin sheath and neurilemma
In Central Nervous • Neurilemma is absent in central nervous system.
System • So, the neuroglial cells called oligodendroglia are responsible for myelinogenesis in
central nervous system

Nueroglial cells
Supporting cell of the nervous system that are non-excitable and do not transmit nerve impulse (action potential).
Six types of neuroglial cells
332
Medico Express Block 2 PHYSIOLOGY
• Four in the CNS
• Two in the PNS
CLASSIFICATION OF NEUROGLIAL CELLS
1. Central Neuroglial cells
Nueroglial cells Function
1. Astrocytes • Twist around the nerve cells and form the supporting network in brain and spinal cord
• Form the blood-brain barrier and thereby regulate the entry of substances from blood
into brain tissues
• Maintain the chemical environment of ECF around CNS neurons
• Provide calcium and potassium and regulate neurotransmitter level in synapses
• Regulate recycling of neurotransmitter during synaptic transmission
2. Microglial • Engulf and destroy the microorganisms and cellular debris by means of phagocytosis
• Migrate to the injured or infected area of CNS and act as miniature macrophages.
3.Oligodendrocytes • Provide myelination around the nerve fibers in CNS where Schwann cells are absent
• Provide support to the CNS neurons by forming a semi-stiff connective tissue between
the neurons.
4. Ependymal cells • Line the central cavity of the spinal cord and brain
• Bear cilia – Help circulate the CSF (cerebrospinal fluid)

2. Peripheral neuroglial cells.


Nueroglial cells Function
Schwann cells: • Provide myelination (insulation) around the nerve fibers in PNS
• Play important role in nerve regeneration.
• Remove cellular debris during regeneration by their phagocytic activity.
Satellite cells • Provide physical support to the PNS neurons
• Help in regulation of chemical environment of ECF around the PNS neurons

SUMMARY
1. Neuron Structure:
o Composed of nerve cell body (soma) with nucleus, Nissl bodies (protein synthesis), neurofibrils
(cytoskeleton), mitochondria, and Golgi apparatus.
o Dendrites are short, branched processes carrying impulses toward the cell body.
o Axon is a single long process transmitting impulses away from the cell body, lacking Nissl bodies.
2. Axonal Transport:
o Microtubules and motor proteins (dynein and kinesin) enable transport.
o Orthograde transport: from cell body to axon terminal (fast and slow).
o Retrograde transport: from axon terminal to cell body, recycling vesicles and transporting
signaling molecules.
3. Myelin Sheath Formation (Myelinogenesis):
o In PNS, formed by Schwann cells wrapping axons; begins prenatally and completes after birth.
o In CNS, oligodendrocytes form myelin, with no neurilemma present.
4. Neuroglial Cells:
o Non-excitable supporting cells; 4 types in CNS (astrocytes, microglia, oligodendrocytes, ependymal
cells), 2 in PNS (Schwann cells, satellite cells).
o Functions include support, myelination, phagocytosis, blood-brain barrier formation, and chemical
environment regulation.

333
Medico Express Block 2 PHYSIOLOGY

MS-P-006 Medical Physiology (Classification of Neurons & Fibers) Learning Objectives


Classify neurons functionally
Classify nerve fibers according to Erlanger & Gasser Classification

CLASSIFICATION OF NUERONS DEPENDING UPON THE FUNCTION


1.Motor or efferent • Neurons which carry the motor impulses from central nervous system to peripheral effector
neurons organs like muscles, glands, blood vessels, etc.
• Generally, each motor neuron has a long axon and short dendrites
2. Sensory or afferent • Neurons which carry the sensory impulses from periphery to central nervous system.
neurons. • Generally, each sensory neuron has a short axon and long dendrites.

CLASSIFICATION OF NERVE FIBERS DEPENDING UPON ERLANGER-GASSER CLASSIFICATION


Erlanger and Gasser classified the nerve fibres into three major
types on the basis of: EXPRESS HIT
On the basis of diameter (thickness) of the fibers;
1- Type A nerve fibers Type Aα fibers (or Type I): Largest diameter, fastest
2- Type B nerve fibers conduction (~80–120 m/s); carry proprioception and
3- Type C nerve fibers somatic motor signals.
Type A nerve fibers are divided into four types: Type Aβ fibers (or Type II): Slightly smaller; conduct
• Type A alpha or Type I nerve fibers touch and pressure signals.
• Type A beta or Type II nerve fibers Type Aγ fibers: Motor fibers to muscle spindles
• Type A gamma nerve fibers (intrafusal fibers); involved in muscle tone regulation.
• Type A delta or Type III nerve fibers Type Aδ fibers (or Type III): Smallest of Type A;
1. Diameter (thickness) of the fibres conduct fast pain, temperature, and crude touch.
Among these fibres, type A nerve fibres are the thickest fibres and
type C nerve fibres are the thinnest fibres

PAST SEQ: A research student of neurophysiology did nerve conduction studies on a subject. He found that the conduction velocities
are different in nerves. (Annual 2012)
Which types of nerve fibers have fastest and slowest conduction velocities?

2. Velocity of conduction of impulse


Velocity of impulse through a nerve fibre is directly proportional to the thickness of the fibre.
SUMMARY
1. Classification of Neurons by Function:
o Motor (efferent) neurons carry impulses from CNS to effectors, typically with long axons and short
dendrites.
o Sensory (afferent) neurons carry impulses from periphery to CNS, usually with long dendrites and
short axons.
2. Classification of Nerve Fibers (Erlanger-Gasser):
o Divided into Type A (thickest), B, and C (thinnest) fibers based on diameter.
o Type A fibers subdivided into alpha, beta, gamma, and delta; conduction velocity is proportional to
fiber thickness.

334
Medico Express Block 2 PHYSIOLOGY

MS-P-007 Medical Physiology (Action Potential of Neurons) Learning Objectives


Define Action Potential
Enlist the Properties of action potential
Describe the ionic basis of an action potential
Explain the phases of action potential
Explain the effects of hyperkalemia and Hypokalemia on the action potential
Draw monophasic action potential
Explain absolute and relative refractory period

ACTION POTENTIAL
Action potential is defined as a series of electrical changes that occur in the membrane potential when the muscle or nerve is
stimulated.
Properties: Propagative, Long distance signal, both depolarization and repolarization, obeys all or none law, Summation is not
possible, Has refractory period
Ionic Basis of Action Potential
Voltage gated Na+ channels and the voltage gated K+ channels play important role in the development of action potential.
Voltage-gated sodium channel; This channel has two gates, Near outside of channel activation gate and near inside inactivation
gate
• At RMP (-90mV) activation gate is closed which prevent Na+ ions influx and inactivation gate is open.
• On threshold stimulus, membrane potential becomes less negative rising from –90mV toward zero b/w –70 and –50 mV
→ a sudden conformational change in the activation gate and it opens. This is called the activated state and Na+ ions
move inward
• The same increase in voltage that opens the activation gate also closes the EXPRESS HIT
inactivation gate→ but a slower process than opening of activation gate
• The inactivation gate will not reopen until the membrane potential Excessive action potential firing in
returns to or near the original resting membrane potential level hyperkalemia may precipitate cardiac
Voltage-gated potassium channel; One potassium gate (towards inside of arrest before muscle fatigue is even
membrane) clinically apparent.
• At RMP, the gate of the potassium channel is closed and prevent from K+
ions outflux. Hypokalemia-induced action potential
• When membrane potential rises from -90mV towards zero → cause failure is a silent culprit behind muscle
conformational opening of gate and allows K+ ions diffusion outward cramps in diuretic-treated hypertensive
• Potassium channel open at the same time when sodium channels begin patients.
to close
• Decrease in sodium entry to the cell and the simultaneous increase in potassium exit from the cell combine to speed the
repolarization process
Phases of action potential
1. Latent period • No change occurs in the electrical potential immediately after applying the stimulus.
• It is a very short period with duration of 0.5 to 1 millisecond.
• Stimulus artefact: slight irregular deflection of baseline for a very short period.
2. Depolarization • Initial phase of action potential in which inside of the muscle becomes positive and outside
becomes negative.
• Starts after the latent period. Initially, it is very slow and the muscle is depolarized for
about 15 mV
• Firing level and depolarization
the rate of depolarization increases suddenly up to –75 mV
Overshoot
From firing level, the curve reaches isoelectric potential (zero potential) rapidly and then
shoots up (overshoots) beyond the zero potential (isoelectric base) up to +55 mV.
3. Repolarization • The phase of action potential in which the muscle reverses back to the resting membrane
potential.
• That is, within a short time after depolarization the inside of muscle becomes negative and
outside becomes positive.
• When depolarization is completed (+55 mV), the repolarization starts.
• Initially, the repolarization occurs rapidly and then it becomes slow.
• Spike potential
Rapid rise in depolarization and the rapid fall in repolarization are together

335
Medico Express Block 2 PHYSIOLOGY
It lasts for 0.4 millisecond.
• After depolarization or negative after potential
Rapid fall in repolarization is followed by a slow repolarization.
Its duration is 2 to 4 milliseconds.
• After hyperpolarization or positive after potential
After reaching the resting level (–90 mV), it becomes more negative beyond resting level.
This lasts for more than 50 milliseconds.
After this, the normal resting membrane potential is restored slowly.

PAST SEQ: Draw and label the action potential of a nerve fiber?
A student while working on research assignment stimulated a mix nerve and recorded action potential. (Supple 2014)
a. Name the type of action potential.
b. What are physiological basis of production of this action potential?

Effect of Hyperkalaemia and hypokalaemia on Nernst potential of nerves


Hyperkalaemia • It is a condition characterized by abnormally high levels of potassium in the blood.
• Elevated potassium levels can cause depolarization of the membrane potential, reducing the driving
force for potassium efflux, and causing the Nernst potential for potassium ions to become less
negative.
• This can increase the excitability of nerves, making them more likely to fire action potentials and
potentially leading to muscle weakness, paralysis, and cardiac arrhythmias.
Hypokalaemia • It is a condition characterized by abnormally low levels of potassium in the blood.
• Reduced potassium levels can cause hyperpolarization of the membrane potential, increasing the
driving force for potassium efflux, and causing the Nernst potential for potassium ions to become
more negative.
• This can decrease the excitability of nerves, making them less likely to fire action potentials and
potentially leading to muscle weakness, paralysis, and cardiac arrhythmias.
Monophasic action potential
A monophasic action potential is a recorded waveform of an action potential from a single area of excitable tissue (e.g., cardiac or
skeletal muscle), where only one polarity (usually upward or positive) is seen.

336
Medico Express Block 2 PHYSIOLOGY
Refractory period EXPRESS HIT
• Refractory period is the period at which the nerve does not show any response to a
stimulus. The refractory period sets a
• It is because already one action potential is in progress in the nerve during this period. natural upper limit on neuronal
• The nerve is unexcitable to further stimulation until it is repolarized. firing rate, forming the
• Refractory period is of two types. biological ceiling for signal
1. Absolute Refractory Period: the period during which the muscle does not show any encoding speed.
response at all, whatever may be the strength of stimulus.
2. Relative Refractory Period: the period, during which the nerve shows some response if the strength of stimulus is increased
to maximum.
PAST SEQ: Absolute Refractory Period (ARP).
What is refractory period, give its types?
SUMMARY
1. Definition and Properties:
Action potential is a rapid electrical change in membrane potential of nerve/muscle upon stimulation. It
propagates over long distances, follows the all-or-none law, cannot summate, and has a refractory period.
2. Ionic Basis:
Voltage-gated Na⁺ channels (with activation and inactivation gates) open on threshold causing Na⁺ influx
(depolarization). Voltage-gated K⁺ channels open later, allowing K⁺ efflux (repolarization), restoring resting
potential.
3. Phases:
o Latent period (no electrical change),
o Depolarization (inside becomes positive, overshoot to +55 mV),
o Repolarization (returns to negative resting potential),
o Afterhyperpolarization (membrane potential briefly more negative than rest).
4. Refractory Period and Effects of Potassium Levels:
o Absolute refractory period: no response to stimulus.
o Relative refractory period: response possible with strong stimulus.
o Hyperkalaemia reduces negativity of Nernst potential, increasing excitability; hypokalaemia
increases negativity, reducing excitability.

337
Medico Express Block 2 PHYSIOLOGY

PAST MCQS
1. Endoplasmic reticulum (ER) is involved in the synthesis of d) A line
which of the following? e) M-line
a) Lipid Correct Answer: a)
b) Protein In sarcomere contraction, the A band remains relatively constant.
c) Carbohydrate 7. Which of the following is an example of isometric contraction?
d) Amino acid a) Squats
e) Nucleotide b) Stair climbing
Correct Answer: a) c) Push-ups
SER is involved in the synthesis of lipids, including cholesterol and d) Running
phospholipids, which are used in the production of new e) Lifting weights
cellular membranes. In cells of the liver, SER contributes to the Correct Answer: c)
detoxification of drugs and harmful chemicals. Push-ups. Isometric contraction occurs when muscle length
2. During depolarization, which of the following can occur? remains relatively constant as tension is produced.
a) Na-Ca influx 8. Which of the following is involved in axon reflex?
b) Na-K efflux a) Actin
c) Na-K efflux b) Myosin
d) Na-Ca influx c) Collagen
e) Ca-K efflux d) Tropomyosin
Correct Answer: a) e) Troponin
During depolarization, the membrane potential rapidly shifts from Correct Answer: a)
negative to positive. As sodium ions rush back into the cell, Axon reflexes involve the activation of a C-fiber, which conducts
they add positive charge to the cell interior, changing the information to the CNS as well as to other branches of the
membrane potential from negative to positive. stimulated afferent nerve ending.
3. Inside the cell membrane, low K ion concentration causes 9. Myosin phosphate is involved in which of the following
which of the following effects? processes?
a) Excess -ve a) Contraction
b) Excess +ve b) Relaxation
c) Excess positive and negative c) Stress
d) Neutral d) Emergency condition
e) None of these e) Sleeping
Correct Answer: a) Correct Answer: b)
Inside the cell membrane, low K ion concentration causes an excess Myosin light chain phosphatase is responsible for the
of negative charge (excess -ve), which is termed dephosphorylation of the regulatory light chain of the motor
repolarization, and if done excessively, it is termed protein myosin-II, negatively regulating actomyosin-based
hyperpolarization. contractility.
4. Which of the following defines chronaxie? 10. If a nerve fiber is put into a sodium-free solution, what will be
a) Half of rheobase the change in action potential?
b) Double the rheobase a) It will increase
c) Double of utilization time b) It will decrease
d) Equal to rheobase c) It will remain the same
e) Four times rheobase d) It will increase first then decrease
Correct Answer: a) Half of rheobase e) It will decrease than increase
Explanation: Chronaxie is defined as the minimum time required to Correct Answer: c
stimulate a nerve or muscle fiber with a current that is If a nerve fiber is put into a sodium-free solution, the action
twice the rheobase. Rheobase, on the other hand, is potential will remain the same.
the minimum current strength of infinite duration 11. What is the primary cause of the resting membrane potential?
required to elicit a response. Therefore, chronaxie is a) Efflux of potassium
related to the time aspect of stimulation at a current b) Influx of potassium
strength of double the rheobase, not half of it. c) Efflux of sodium
5. Ca influx causes the release of which of the following? d) Influx of sodium
a) Heparin e) Influx of calcium
b) Serotonin Correct Answer: a
c) Bradykinin Explanation: The resting membrane potential is primarily
d) Acetylcholine established by the outflow (efflux) of potassium ions, driven
e) E-dopamine by the concentration and electrical gradients.
Correct Answer: d) 12. How do chemicals or action potentials spread between unitary
Calcium influx causes the release of acetylcholine. smooth muscles?
6. In the contraction of the sarcomere, there is no change in a) Tight junctions
which of the following structures? b) Desmosomes
a) A band c) Gap junctions
b) I band d) Anchoring junctions
c) B zone e) All of these
369
Medico Express Block 2 PHYSIOLOGY
Correct Answer: c Rigor mortis occurs due to the failure of detachment of myosin
Gap junctions facilitate the rapid exchange of small molecules and heads from actin filaments in muscle fibers after death.
play a vital role in intercellular communication among unitary 19. Neurotransmitters are released by an influx of which ion into
smooth muscle cells. the nerve?
13. What is the term for the functional organization of muscle a) Ca
fibers within a muscle, with a single motor neuron innervating b) K
all the muscle fibers? c) Na
a) Neuronal plexus d) Ba
b) Neuromuscular unit e) Cl
c) Motor unit Correct Answer: a
d) Sensory unit Neurotransmitters are released by the influx of calcium ions into
e) Myenteric plexus the nerve terminal.
Correct Answer: c 20. Resting membrane potential is primarily due to the:
A motor unit consists of a motor neuron and all the muscle fibers it a) K efflux
innervates, working together to coordinate muscle b) Na efflux
contractions. c) Na influx
14. Which event is least likely to occur during the pharyngeal stage d) K influx
of swallowing? e) Ca efflux
a) Deglutition apnea Correct Answer: a
b) Closure of the posterior nares The resting membrane potential is primarily determined by the
c) Narrowing of the palatopharyngeal folds efflux of potassium ions, driven by concentration and electrical
d) Opening of the glottis gradients.
e) Closure of the glottis 21. During which process does the I band shorten?
Correct Answer: d a) Muscle contraction
During the pharyngeal stage of swallowing, opening of the glottis is b) Muscle relaxation
unlikely to occur as it would risk airway obstruction. c) Muscle tetanization
15. What functions are associated with the contraction of the d) Muscle scale intact
gallbladder and the stimulation of enzymerich pancreatic e) Muscle hyperpolarization
secretion? Correct Answer: a
a) Cholecystokinin During muscular contraction, the myosin heads pull the actin
b) Gastrin filaments toward each other, resulting in a shortened
c) Motilin sarcomere. While the I band and H zone will disappear or
d) Secretin shorten, the A band length remains unchanged. The I band
e) Substance corresponds to the region of actin that does not overlap with
Correct Answer: a myosin.
Cholecystokinin stimulates the contraction of the gallbladder and 22. How does intracellular fluid differ from extracellular fluid?
the release of bile into the intestine, as well as the secretion a) Increased quantity of potassium ions
of enzymes from the pancreas. b) Increased quantity of sodium ions
16. Why does cardiac muscle act as a functional syncytium? c) Increased quantity of phosphate ions
a) Consists of single nucleated cells d) Decreased quantity of potassium ions
b) Has a long refractory period e) Decreased quantity of phosphate ions
c) Has larger T-tubules Correct Answer: a
d) Has gap junctions between adjacent cells Intracellular fluid generally has a high concentration of potassium
e) Consists of multiple nucleated cells and a low concentration of sodium, while extracellular fluid
Correct Answer: d has the opposite composition with a high sodium
Cardiac muscle acts as a functional syncytium due to the presence concentration and low potassium concentration.
of gap junctions, allowing rapid, coordinated contraction along 23. What characterizes isometric muscle contraction?
its entire length. a) ATP is utilized in large amounts
17. Among the following, which structure has the highest ionic b) Muscle length decreases during the contraction
permeability? c) Much work is performed by the muscle
a) Gap junction d) There is stretching of the elastic component
b) Interneuronal junction e) Elastic component is intact
c) Neuromuscular junction Correct Answer: d)
d) Nodes of Ranvier Short Explanation:
e) All have the same permeability In isometric contraction, the muscle doesn’t shorten, but the
Correct Answer: d internal (elastic) components stretch to generate tension.
Nodes of Ranvier have the highest ionic permeability as they are 24. Which muscle protein covers the active site on actin filaments
unmyelinated and allow for swift conduction of nerve during rest?
impulses. a) Actinin
18. What is the physiological process behind rigor mortis? b) Myosin
a) Detachment of myosin head c) Tropomyosin
b) Failure of detachment of myosin head d) Troponin-T
c) Breakdown of actin e) Troponin-I
d) Attachment of myosin head Correct Answer: c
e) Breakdown of troponin Tropomyosin covers the active site on actin filaments before
Correct Answer: b muscle contraction. When it is removed, actin and myosin
heads can bind.

370
Medico Express Block 2 BIOCHEMISTRY

BIOCHEMISTRY
MS-B-001 Biochemistry Classification carbohydrates Learning Objectives
Classify carbohydrates along with the structure and biomedical importance of each class

CARBOHYDRATES
Definition: Carbohydrates are defined chemically as aldehyde or ketone derivatives of the higher polyhydric alcohols, or
compounds that yield these derivatives on hydrolysis.
Classification:
Carbohydrates can be classified into four major groups: monosaccharides, disaccharides, oligosaccharides, and polysaccharides.
Each class has its unique structure and biomedical importance:
1. Monosaccharides:
• Monosaccharides, or simple sugars, are the simplest form of carbohydrates that cannot be hydrolyzed further
into simpler forms.
• They have the general formula CnH2nOn.
• Examples:
− Trioses (e.g., glyceraldehyde),
− Tetroses (e.g., erythrose),
− Pentoses (e.g., ribose),
− Hexoses (e.g., glucose).
• Biomedical Importance: Monosaccharides serve as a chief energy source in the body. They are constituents of
compound lipids and conjugated proteins. Certain carbohydrate derivatives are also used as drugs, and
monosaccharides like glucose are crucial for various metabolic pathways.

2. Disaccharides:
• Disaccharides are sugars that yield two molecules of the same or different monosaccharides upon hydrolysis.
• They have the general formula Cn(H2O)n-1.
• Examples:
− Maltose (two glucose molecules), EXPRESS HIT
− Lactose (glucose and galactose),
− Sucrose (glucose and fructose). Lactose is the only disaccharide
• Biomedical Importance: Disaccharides are important sources of of animal origin, significant in
energy, and their degradation products play roles in various metabolic infant nutrition. It deficiency
pathways. For example, lactose is the principal sugar in milk, and its leads to lactose intolerance
breakdown products are utilized for the synthesis of other substances.

PAST SEQ: Write about lactose and its biological importance.


Q1. Define disaccharides. Give three examples. Write down their product of their hydrolysis.
Q4. What are oligosaccharides? Give their role.

3. Oligosaccharides:
• Oligosaccharides are sugars that yield 3 to 10 monosaccharide units upon hydrolysis.
• Example: Maltotriose.
• Biomedical Importance: Oligosaccharides are less common but still play roles in various biological processes,
although their specific biomedical importance may vary.

383
Medico Express Block 2 BIOCHEMISTRY
4. Polysaccharides (Glycans):
• Polysaccharides are sugars that yield more than ten molecules of monosaccharides upon hydrolysis.
• They have the general formula (C6H10O5)n.
• Polysaccharides are further classified into homopolysaccharides (homoglycans) and heteropolysaccharides
(heteroglycans).
• Examples:
i. Homopolysaccharides include
1. Starch,
2. Glycogen,
3. Cellulose (β-D-glucose units linked by β-1,4-glycosidic bonds),
4. Dextrins,
ii. Heteropolysaccharides are mucopolysaccharides (e.g., glycosaminoglycans).
• Biomedical Importance: Polysaccharides serve as energy storage molecules (e.g., glycogen in animals, starch in
plants), and structural components (e.g., cellulose in plants), and play roles in cell-cell recognition and signaling.

PAST SEQ: Q1. Define homopolysaccharides. Name 4 of them with one major function they perform
Give the structures and biochemical role of cellulose.
i) Biochemical mechanism:
Cellulose is an indigestible fiber that absorbs water in the intestine, increases stool bulk, and stimulates peristalsis, relieving
constipation and improving bowel movements in IBS.
ii) Structure of cellulose:
Cellulose is a linear polymer of β-D-glucose units linked by β-1,4-glycosidic bonds, forming rigid fibers due to hydrogen
bonding.
iii) Hemicellulose:
Hemicellulose is a branched heteropolysaccharide made of various sugars; it is partially fermentable and adds bulk to stool
like cellulose.

SUMMARY
Definition:
Carbohydrates are aldehyde or ketone derivatives of polyhydric alcohols, or compounds that yield them on
hydrolysis.
Classification:
1. Monosaccharides:
Simple sugars (e.g., glucose, ribose); not hydrolyzable; provide energy and participate in metabolism and
structural roles.
2. Disaccharides:
Yield two monosaccharides on hydrolysis (e.g., maltose, lactose, sucrose); serve as energy sources and have
metabolic significance.
3. Oligosaccharides:
Yield 3–10 monosaccharides (e.g., maltotriose); less common but important in biological recognition and
processes.
4. Polysaccharides:
Yield >10 monosaccharides (e.g., starch, glycogen, cellulose); classified as homo- or heteropolysaccharides;
function in energy storage, structure, and signaling.

384
Medico Express Block 2 BIOCHEMISTRY

MS-B-002 Biochemistry (Carbohydrates) Learning Objectives


Explain the isomerization of carbohydrates

ISOMERIZATION
The isomerization of carbohydrates involves the conversion of one form of a carbohydrate molecule into another form, typically
involving changes in the spatial arrangement of atoms around specific carbon atoms. This process is governed by several principles
outlined in the provided text:
Asymmetric Carbon and Stereoisomerism:
• Carbohydrates contain asymmetric carbon atoms, where four different atoms or groups of atoms are attached
to a single carbon atom.
• The presence of asymmetric carbon atoms leads to the formation of stereoisomers, which are compounds with
identical molecular formulas but different spatial arrangements of atoms.

D-Series and L-Series:


• The orientation of hydrogen (H) and hydroxyl (OH) groups around the asymmetric carbon atoms determines
whether the sugar molecule belongs to the D-series or L-series.
• Most monosaccharides occurring in mammals are D-sugars.

Optical Activity:
• Asymmetric carbon atoms confer optical activity on carbohydrate molecules.
• Dextrorotatory compounds rotate plane-polarized light to the right, while laevorotatory compounds rotate it to
the left.
Anomers and Mutarotation:
• When carbohydrate molecules undergo cyclization, they form two cyclic compounds known as anomers.
• An anomeric carbon atom is created during cyclization.
• Mutarotation refers to the phenomenon where the optical rotation of a carbohydrate solution changes over
time due to the conversion between different anomeric forms.
• This process involves the equilibrium between the alpha and beta forms of the carbohydrate.

PAST SEQ: Give a comparison between: (Supple 2016)


i) Anomer and Epimer

385
Medico Express Block 2 BIOCHEMISTRY
Comparison between Anomer and Epimer
Feature Anomer Epimer
Definition Anomers are a special type of epimers that differ at Epimers are stereoisomers that differ in
the anomeric carbon (C-1 in aldoses, C-2 in ketoses). configuration at only one carbon other than the
anomeric carbon.
Carbon Atom Anomeric carbon (formed during cyclic structure Any carbon except the anomeric carbon
Involved formation)
Type of Isomerism A subtype of epimerism specific to cyclic sugars A broader category of diastereomers
Example α-D-glucose and β-D-glucose (differ at C-1) D-glucose and D-mannose (differ at C-2), D-
glucose and D-galactose (C-4)
Occurs In Only in cyclic (hemiacetal/hemiketal) sugars Can occur in both cyclic and acyclic
monosaccharides
Interconversion Interconvert via mutarotation in solution Do not undergo mutarotation
Haworth Projection and Epimers:
• Carbohydrates in their cyclic forms can be represented using Haworth projections, which depict the spatial
arrangement of atoms within the cyclic ring.
• Epimers are sugars that differ from each other only in the configuration around a single carbon atom.
• Epimerization is the process by which one epimer is converted into another, typically catalyzed by enzymes
called epimerases.

PAST SEQ: Q8. Discuss the structures and isomerism in carbohydrates taking glucose as reference sugar.
b) What are epimers? Explain the epimers of glucose. (Supply 2020 held in 2021)

SUMMARY
Asymmetric Carbon & Stereoisomerism:
Carbohydrates have asymmetric carbon atoms, giving rise to stereoisomers with the same molecular formula but
different spatial arrangements.
D- & L-Series:
Determined by the position of –OH and –H around asymmetric carbon; most sugars in mammals are D-series.
Optical Activity:
Carbohydrates rotate plane-polarized light; dextrorotatory sugars rotate it right, laevorotatory rotate it left.
Anomers & Mutarotation:
Cyclization creates anomers (α and β forms); mutarotation is the change in optical rotation due to interconversion
between these forms.
Haworth Projection & Epimers:
Cyclic structures are shown in Haworth projections. Epimers differ at one carbon; epimerization is enzyme-
catalyzed conversion between them.

386
Medico Express Block 2 BIOCHEMISTRY

PAST MCQS
1. Hydroxylation of vitamin D occurs at two sites within the body, Correct Answer: B
which possibly are: HbA1c is a form of hemoglobin used to measure average blood
a) Stomach and liver glucose levels over a period of approximately three months.
b) Liver and pancreas 7. Bilirubin bound to glucuronic acid will increase in which
c) Kidney and stomach pathology:
d) Liver and kidney a) Hepatic jaundice
e) Brain and kidney b) Prehepatic jaundice
Correct Answer: D c) Posthepatic jaundice
The liver converts vitamin D to 25OHD. The kidney converts 25OHD d) Pellagra
to 1,25(OH)2D and 24,25(OH)2D. Other tissues contain these e) Dermatitis
enzymes, but the liver is the main source for 25-hydroxylation, Correct Answer: C
and the kidney is the main source for 1α-hydroxylation. In posthepatic jaundice, there is a block in the outward transport of
2. Functional form of vitamin D in the body is: bilirubin, causing the amount of conjugated bilirubin to
a) Cholecalciferol increase in the liver.
b) 25-Hydroxycholecalciferol 8. Oxygen molecules attach to which form of iron only:
c) Ergocalciferol a) Fe2+
d) 1,25-dihydroxycholecalciferol b) Fe3+
e) Tocopherol c) Copper
Correct Answer: D d) K+
The functional form of vitamin D in the body is 1,25- e) Calcium
dihydroxycholecalciferol, also known as calcitriol. Correct Answer: A
3. Demineralization of bones resulting in rickets in children and Oxygen binds to iron in its ferrous (Fe2+) form. The ferric form
osteomalacia in adults is secondary to the deficiency of: (Fe3+) must be reduced to the ferrous form for efficient
a) Vitamin C oxygen binding, as the ferric form would result in difficulties
b) Vitamin D with unloading oxygen, leading to conditions like
c) Vitamin E methemoglobinemia. This can cause a condition known as
d) Vitamin K "chocolate cyanosis."
e) Vitamin A 9. Which structure of proteins has the ability to form domains in
Correct Answer: B itself:
Not enough vitamin D makes it difficult to maintain proper calcium a) Primary
and phosphorus levels in bones, which can cause rickets in b) Secondary
children and osteomalacia in adults. c) Tertiary
4. Vitamin D can be manufactured endogenously via which d) Quaternary
phospholipid: e) All of these
a) Phospholipid Correct Answer: C
b) Cardiolipin A protein domain is a region of a protein's polypeptide chain that is
c) Sphingophospholipids self-stabilizing and can fold independently from the rest of the
d) Cholesterol protein. Tertiary structure is formed by further folding and
e) None of the above interaction of secondary structures, creating a three-
Correct Answer: D dimensional conformation.
In the presence of sunlight, a form of vitamin D3 called 10. pH of protein occupying the plasma can possibly be:
cholecalciferol is synthesized from a derivative of the steroid a) 7.3 to 7.45
cholesterol in the skin. The liver converts cholecalciferol to b) 6.5 to 7
calcidiol, which is then converted to calcitriol (the active c) 7 to 8
chemical form of the vitamin) in the kidneys. d) 5 to 6
5. Adequate sunlight can be a good source for which of the e) 6 to 7
following: Correct Answer: A
a) Vitamin A The pH of proteins in the plasma is typically within the range of 7.3
b) Vitamin K to 7.45.
c) Vitamin D 11. A 63-year-old patient was diagnosed with Alzheimer's disease
d) Vitamin E after repeated complaints of memory loss and diminished
e) Vitamin B1 motor functions. Malformation of which part of the protein
Correct Answer: C structure is the cause:
Adequate sunlight exposure is a good source of vitamin D. a) Beta-pleated sheet
6. A patient of 40 years old comes to OPD with the concern of b) Alpha helical structure
imbalanced sugar levels. A particular form of hemoglobin is c) Gamma globulin denaturation
used to calculate sugar levels concerning up to 3 months, d) Hydroxyl bond broken
which is: e) Any of these
a) HbA1e Correct Answer: A
b) HbA1c In Alzheimer's disease, there is an accumulation of beta-amyloid
c) HbA1u proteins, leading to the malformation of betapleated sheets in
d) Hb-glucose neurological structures. This accumulation disrupts normal
e) None of these
413
Medico Express Block 2 BIOCHEMISTRY
brain function, resulting in memory loss and motor Aspartate is not a positively charged amino acid; it is negatively
dysfunction. charged. Histidine, arginine, and lysine are positively charged
12. Proteins responsible for maintaining plasma osmotic pressure amino acids.
are: 18. A very high-strength (voltage) electrophoresis is used for the
a) Globulin separation of:
b) B-Albumin a) Proteins
c) Gamma globulin b) Lipoproteins
d) Elastin c) Amino acids
e) Fibrin d) Lipids
Correct Answer: B e) Nucleotides
B-Albumin plays a significant role in maintaining plasma osmotic Correct Answer: C
pressure by helping to balance the oncotic pressure of plasma High-voltage electrophoresis is used for the rapid separation and
proteins, which in turn prevents excessive fluid from leaking identification of small amphoteric molecules, such as amino
out of capillaries into tissues. acids.
13. All of the following can occupy the alpha helix structure of 19. Types of bonds in the Quaternary structure of protein are:
proteins EXCEPT: a) Peptide bonds
a) Proline b) Disulfide bonds
b) Histidine c) Covalent bonds
c) Lysine d) Non-covalent bonds
d) Leucine e) Hydrogen bonds
e) E-Tyrosine Correct Answer: D
Correct Answer: A The quaternary structure of proteins is held together by non-
Proline contains a secondary amino group, making it an imino acid. covalent bonds, including hydrophobic and hydrophilic
Proline disrupts the alpha helix structure due to its unique interactions, as well as salt linkages between acidic and basic
structure, which introduces kinks in the protein chain. side chains.
14. Absence of uroporphyrinogen-III decarboxylase leads to the 20. In the case of immunoglobulins, binding occurs at the:
accumulation of a substrate in the pathway. Possible a) Variable regions
accumulation of uroporphyrin is possible, this condition is b) Constant regions
called: c) Polar regions
a) Porphyria cutanea tarda d) Non-polar regions
b) Lead poisoning e) All of these
c) Acute intermittent porphyria Correct Answer: A
d) Variegate porphyria Binding of immunoglobulins (antibodies) occurs at the variable
e) None of the above regions, specifically on the ends of the light chains. These
Correct Answer: A variable regions are responsible for binding to antigens.
Absence of uroporphyrinogen-III decarboxylase can lead to the 21. Amino acid capable of performing buffering action:
accumulation of uroporphyrin, resulting in the condition a) Histidine
known as Porphyria cutanea tarda. b) Arginine
15. Immunoglobulins responsible for acute allergic responses is: c) Proline
a) IgG d) Lysine
b) IgD e) Leucine
c) IgA Correct Answer: A
d) IgE Histidine is an amino acid capable of performing buffering action. It
e) IgM is present in proteins and can act as a buffer by accepting or
Correct Answer: D donating protons, helping to maintain pH stability.
IgE (Immunoglobulin E) is responsible for acute allergic responses. 22. Proteins can be separated from each other by what technique:
When allergens enter the body, IgE antibodies play a crucial a) Filtration
role in triggering the release of histamine, bradykinin, and b) Chromatography
other substances that lead to allergy symptoms. c) Ion exchange
16. Albumin maintains plasma osmotic pressure via the ability of: d) Crystallization
a) A-Low molecular weight e) Identification
b) B-High molecular weight Correct Answer: C
c) C-Low molecular mass Ion exchange chromatography is commonly used to separate
d) D-High molecular mass proteins, peptides, amino acids, or nucleotides based on their
e) E-Low molecular density charge properties. Amino acids contain both positively and
Correct Answer: A negatively charged groups, making this technique suitable for
Albumin maintains plasma osmotic pressure effectively due to its their separation.
low molecular weight. Its small size allows it to occupy more 23. Transmissible encephalopathies are progressively fatal:
spaces and efficiently balance oncotic pressure, preventing a) Prion disease
edema. b) Collagen disease
17. ALL are positively charged amino acids except: c) Amyloid disease
a) Aspartate d) Elastin disease
b) Histidine e) None of these
c) Arginine Correct Answer: A
d) Lysine Transmissible encephalopathies are caused by prions, which are
e) Leucine misfolded proteins. These diseases are progressively fatal and
Correct Answer: A

414
Medico Express Block 2 PATHOPHYSIOLOGY & PHARMACOTHERAPEUTICS

PHARMACOTHERAPEUTICS
MS-Ph-01 Pharmacology & Therapeutics (Drugs acting on Neuromuscular Junction (NMJ)) Learning
Objectives
Explain the mechanism by which drugs can stimulate NMJ.
Explain the mechanism by which drugs can block NMJ

Stimulation of NMJ by Drugs:


• Act as agonists at the nicotinic acetylcholine receptors (nACHRs) located on the motor end plate.
• Mimic the action of acetylcholine
Agonist drugs bind to nACHRs

Activate the receptor

Depolarization of the motor end plate membrane

Triggers the opening of voltage-gated ion channels

Influx of sodium ions (na⁺) and efflux of potassium ions (K⁺).

Change in membrane potential prpagates an action potential along the muscle fiber

Muscle contraction

• Examples: succinylcholine and nicotine.


PAST SEQ: Write a note on neuromuscular junction stimulation. And types of NMJ blockers.
Blocking NMJ by Drugs:
• As antagonists at the nicotinic acetylcholine receptors (nACHRs) located on the motor end plate.
• These drugs prevent the binding of acetylcholine to nACHRs, thereby inhibiting its action and blocking muscle contraction.
• Certain drugs (e.g. botulinum toxin etc.) may indirectly block the NMJ by inhibiting the release of acetylcholine from
presynaptic nerve terminals or by interfering with neuromuscular transmission at other sites.
Types
a. Nondepolarizing Blockers: These drugs competitively inhibit acetylcholine from binding to nAChRs without activating
them → prevent depolarization of the motor end plate and muscle contraction.
Examples: d-tubocurarine and vecuronium.
b. Depolarizing Blockers: These drugs initially activate nAChRs and cause depolarization of the motor end plate, similar to
acetylcholine.
However, unlike acetylcholine, they are resistant to degradation by acetylcholinesterase → prolonged depolarization and
desensitization of nAChRs → muscle paralysis.
Examples: suxamethonium, Succinylcholine EXPRESS HITS
Depolarizing blockers (e.g., succinylcholine):
• Phase I: Persistent depolarization
(fasciculations)
• Phase II: Desensitization block (flaccid
paralysis)
• Not reversed by acetylcholinesterase
inhibitors; may worsen Phase II block.

429
Medico Express Block 2 PATHOPHYSIOLOGY & PHARMACOTHERAPEUTICS

SUMMARY
1. Stimulation of NMJ:
• Drugs act as agonists at nicotinic acetylcholine receptors (nAChRs) on the motor end plate.
• Mimic acetylcholine (ACh) and promote muscle contraction.
• Examples: Succinylcholine, nicotine.
2. Blocking the NMJ:
• Drugs act as antagonists at nAChRs, preventing ACh binding and inhibiting muscle contraction.
• Some drugs (e.g., botulinum toxin) indirectly block NMJ by inhibiting ACh release.
Types of NMJ Blockers:
• a. Nondepolarizing Blockers: Competitively inhibit ACh without activating receptors, preventing
depolarization.
o Examples: d-tubocurarine, vecuronium.
• b. Depolarizing Blockers: Activate nAChRs and cause sustained depolarization due to resistance to
acetylcholinesterase.
o Leads to desensitization and paralysis.
o Examples: Succinylcholine (suxamethonium).

430
Medico Express Block 2 PATHOPHYSIOLOGY & PHARMACOTHERAPEUTICS

MS-Ph-02 Pharmacology & Therapeutics (Drugs in Myasthenia Gravis) Learning Objectives


Discuss briefly the therapeutic effect of drugs used in myasthenia gravis.

Myasthenia Gravis
1. Cholinesterase Inhibitors:
• Cholinesterase inhibitors are the mainstay of therapy for myasthenia gravis.
Mechanism of Action:
Inhibiting the enzyme Increase the availability
Enhancing neuromuscular
acetylcholinesterase, which and duration of
transmission and muscle
breaks down acetylcholine acetylcholine at the
contraction.
in the synaptic cleft. neuromuscular junction

Examples:
− Pyridostigmine
− Neostigmine
2. Immunosuppressant Drugs
• control the autoimmune response by suppressing the production of antibodies that target the nicotinic
acetylcholine receptors at the neuromuscular junction.
Examples :
− Steroids (e.g., Prednisone),
− Cyclosporine,
− Azathioprine.
3. Diagnostic Agents (e.g., edrophonium):
• Edrophonium is used as a diagnostic test for myasthenia gravis to assess neuromuscular function.
• It is a short-acting cholinesterase inhibitor that temporarily improves muscle strength in patients with
myasthenia gravis.
• The response to edrophonium can help differentiate between myasthenic crisis (exacerbation of myasthenia
gravis) and cholinergic crisis (excessive drug therapy).
PAST SEQ: Write names of drugs used in myasthenia gravis.

SUMMARY
1. Cholinesterase Inhibitors
o Main treatment for myasthenia gravis.
o Mechanism: Inhibit breakdown of acetylcholine, enhancing neuromuscular transmission.
o Examples: Pyridostigmine, Neostigmine.
2. Immunosuppressant Drugs
o Suppress the autoimmune attack on nicotinic acetylcholine receptors.
o Examples: Prednisone (steroid), Cyclosporine, Azathioprine.
3. Diagnostic Agent – Edrophonium
o Short-acting cholinesterase inhibitor used for diagnosis.
o Temporarily improves muscle strength in myasthenia gravis.
o Helps differentiate myasthenic crisis from cholinergic crisis.

431
Medico Express Block 2 PATHOPHYSIOLOGY & PHARMACOTHERAPEUTICS

PATHOLOGY
MS-Pa-01 Pathology (Muscle remodeling) Learning Objectives
Describe the hyperplasia, hypertrophy, and atrophy of muscle fiber
Explain the histopathological basis of leiomyoma

Muscle Remodeling:
Aspect Hyperplasia Hypertrophy Atrophy
Definition Increase in the number of Increase in the size of individual Shrinkage in the size of muscle cells
muscle cells muscle cells
Physiological Rare Common Rare
Stimuli Hormonal changes, Increased functional demand, Decreased workload, loss of innervation, diminished
compensatory growth factors/hormones blood supply, inadequate nutrition, loss of endocrine
mechanisms stimulation, aging
Examples Hormonal hyperplasia Skeletal muscle hypertrophy due to Muscle disuse atrophy due to immobilization, loss of
during puberty or strength training or weightlifting innervation in neurological conditions, senile atrophy
pregnancy due to aging
Mechanism Increased proliferation of Enlargement of individual muscle Decreased protein synthesis, increased protein
muscle cells cells degradation, activation of autophagy
Cellular Increase in the number of Increase in the size of myofibrils Reduction in myofibril size, increase in autophagic
Changes muscle cells and organelles vacuoles
PAST SEQ: A weightlifter notices that their muscles have increased in size after months of training. Describe the cellular
mechanism behind this enlargement. What is this muscle adaptation called?
During pregnancy, smooth muscle cells in the uterus multiply. Identify this type of muscle remodeling and describe the
physiological stimulus responsible.

Leiomyoma
Benign tumors develop from smooth muscle fibers. Also known as “fibroids”.
Epidemiology:
• Most common benign tumor in females.
• Affects 30% to 50% of women of reproductive age.
• More frequent in black women.
Genetic and Molecular Factors:
• Rearrangements of chromosomes 6 and 12.
• Mutations in the MED12 gene
434
Medico Express Block 2 PATHOPHYSIOLOGY & PHARMACOTHERAPEUTICS
Hormonal Influence:
• Estrogens and oral contraceptives stimulate leiomyoma growth.
• Tumors tend to shrink post-menopausally.
Morphology:
• Typically sharply circumscribed, firm gray-white masses with a
characteristic whorled cut surface.
• Singly or as multiple tumors scattered within the uterus.
• Variants include
− Intramural,
− Submucosal, and
− Subserosal leiomyomas.
• Histologically characterized by bundles of smooth muscle cells
resembling normal myometrium.
• Foci of fibrosis, calcification, and degenerative softening may be present.

T
PAST SEQ: A 35-year-old woman presents with a firm, well-circumscribed uterine mass. Histology shows bundles of smooth
muscle cells with a whorled pattern. What is the most likely diagnosis?
Clinical Presentation:
• Often asymptomatic and discovered incidentally on routine pelvic examination.
• The most frequent presenting sign is menorrhagia, with or without metrorrhagia (irregular or abnormal bleeding).
Malignant Transformation:
• Leiomyomas rarely, if ever, transform into sarcomas.
• The presence of multiple lesions does not increase the risk of malignancy.
SUMAMRY
Muscle Remodeling:
• Involves hyperplasia (↑ cell number), hypertrophy (↑ cell size), and atrophy (↓ cell size).
• Stimuli include hormones, physical activity, or disuse; mechanisms involve protein synthesis, degradation,
and cell proliferation.
Hyperplasia & Hypertrophy:
• Hyperplasia is rare and hormone-driven (e.g., puberty, pregnancy).
• Hypertrophy is common due to exercise; increases myofibrils and organelle size.
Atrophy:
• Caused by inactivity, nerve loss, poor nutrition, or aging.
• Leads to reduced muscle size via autophagy and decreased protein synthesis.
Leiomyoma (Fibroid):
• A benign smooth muscle tumor, common in reproductive-age women, estrogen-sensitive, and rarely
malignant.
• Often asymptomatic or presents with menorrhagia; has a whorled, firm appearance and may show genetic
mutations (e.g., MED12).

435
Medico Express Block 2 PATHOPHYSIOLOGY & PHARMACOTHERAPEUTICS

MS-Pa-02 Pathology (Diseases of Muscle) Learning Objectives


Describe the histological basis of Duchenne Muscular Dystrophy and myopathy

Duchenne Muscular Dystrophy (DMD)


• DMD is an X-linked muscular dystrophy caused by mutations in the dystrophin gene, resulting in dysfunction of the
dystrophin protein.
Morphology:
• ongoing myofiber necrosis and regeneration, leading to progressive replacement of muscle tissue by fibrosis and fat.
• Cardiac muscles also show variable degrees of myocyte hypertrophy and interstitial fibrosis.

Pathogenesis:
Dysfunction of
dystrophin- Muscle cells
Mutations disrupt the
glycoprotein vulnerable to Myofiber
dystrophin gene on
complex, which membrane tears degeneration.
the X chromosome
stabilizes muscle cells during contraction,
during contraction.

• Cardiac muscle dysfunction also occurs due to dystrophin deficiency, leading to cardiomyopathy.
Clinical Features:
• Initial symptoms → clumsiness and muscle weakness, beginning in the pelvic girdle and progressing to the shoulder
girdle.
436
Medico Express Block 2 PATHOPHYSIOLOGY & PHARMACOTHERAPEUTICS
• Pseudohypertrophy of calves is common due to myofiber hypertrophy initially, followed
by replacement with adipose tissue and fibrosis. EXPRESS HITS
• Cardiac involvement → heart failure and arrhythmias, contributing to mortality. Gowers' sign (using hands
• Cognitive impairment may occur, and high serum creatine kinase levels persist from to rise from squatting) is a
birth. hallmark of proximal
muscle weakness.
Treatment:
• Current treatment primarily involves supportive care.
• Definitive therapy aims to restore dystrophin levels in skeletal and cardiac muscle fibers.
• Genetic approaches, such as anti-sense RNAs to alter RNA splicing or drugs promoting ribosomal "read-through" of stop
codons, are being tested in clinical trials.
Myopathy
Channelopathies, Metabolic Myopathies, and Mitochondrial Myopathies:
i. Ion Channel Myopathies:
• Characterized by inherited defects in ion channels.
• Present with myotonia, hypotonic paralysis episodes, or both.
• Examples include hyperkalemic periodic paralysis and malignant
hyperthermia. EXPRESS HITS
ii. Myopathies due to Inborn Errors of Metabolism: Malignant hyperthermia is due to
• Arise from disorders in glycogen synthesis, lipid handling, or RYR1 gene mutations → uncontrolled
mitochondrial function. Ca²⁺ release → muscle rigidity and
• Include lipid myopathies and disorders of the carnitine transport hypermetabolism.
system.
iii. Mitochondrial Myopathies:
• Stem from mutations in mitochondrial or nuclear genomes.
• Show maternal inheritance and manifest with proximal muscle weakness, ocular involvement, and systemic
manifestations.
Inflammatory Myopathies:
i. Polymyositis:
• Autoimmune disorder with increased MHC class I expression.
• Endomysial inflammatory infiltrates with CD8+ T cells.
• Microscopically: myofiber necrosis and regeneration, responsive to immunosuppressive agents.
ii. Dermatomyositis:
• Often appears as a paraneoplastic disorder in adults.
• Associated with skin manifestations and systemic involvement.
• Microscopically: perivascular mononuclear cell infiltrates and myofiber damage.
iii. Inclusion Body Myositis:
• Common in patients older than 60.
• Features rimmed vacuoles containing protein aggregates.
• Chronic, progressive course, poorly responsive to immunosuppressive agents.
Toxic Myopathies:
i. Thyrotoxic Myopathy:
• Presents with acute or chronic proximal muscle weakness.
• Microscopically: shows myofiber necrosis and regeneration.
ii. Ethanol Myopathy:
• Follows binge drinking episodes, leading to severe rhabdomyolysis.
• Microscopically: exhibits myocyte swelling, necrosis, and regeneration.
iii. Drug Myopathy:
• Can be caused by various medications, such as statins.
• Recognized forms include drug toxicity and immune-mediated myopathy.
SUMMARY
1. Duchenne Muscular Dystrophy (DMD):
o An X-linked disorder caused by mutations in the dystrophin gene, leading to progressive muscle
degeneration and replacement by fat/fibrosis.
o Clinical features: Early pelvic muscle weakness, calf pseudohypertrophy, cardiomyopathy, and
cognitive issues.
o Treatment is primarily supportive; gene therapy approaches are under research.
2. Inherited Myopathies:

437
Medico Express Block 2 PATHOPHYSIOLOGY & PHARMACOTHERAPEUTICS
o Include ion channel myopathies (e.g., periodic paralysis), metabolic myopathies (e.g.,
lipid/glycogen storage disorders), and mitochondrial myopathies (maternal inheritance, systemic
symptoms).
3. Inflammatory Myopathies:
o Polymyositis: Autoimmune with CD8+ T-cell infiltrates.
o Dermatomyositis: Paraneoplastic potential with skin/systemic features.
o Inclusion body myositis: Elderly onset, progressive, with rimmed vacuoles and poor response to
therapy.
4. Toxic Myopathies:
o Caused by thyroid dysfunction, alcohol, or drugs (e.g., statins).
o Characterized by muscle fiber necrosis, regeneration, and in some cases, immune-mediated
damage.

438
Medico Express Block 2 PATHOPHYSIOLOGY & PHARMACOTHERAPEUTICS

PAST MCQS
PHARMACOLOGY
1. Local anesthetics exert their effect by interfering with the muscle contraction without initially causing muscle twitching
transmission of nerve impulses. Which of the following (fasciculations).
cellular structures or mechanisms is the primary target of 5. Certain local anesthetics are applied directly to the surface of
local anesthetic drugs? (Annual 2024) tissues to provide localized numbness. Which of the following
a) Acetylcholine receptors drugs is a known surface-acting anesthetic? (Annual 2024)
b) Potassium channels a) Lidocaine
c) Sodium channels b) Procaine
d) Calcium channels c) Bupivacaine
Answer: c) Sodium channels d) Cocaine
Explanation: Local anesthetics primarily block voltage-gated Answer: d) Cocaine
sodium channels in nerve cell membranes. This blockage Explanation: Cocaine was one of the first local anesthetics
prevents the influx of sodium ions necessary for the discovered and is unique in that it also has vasoconstrictive
depolarization phase of the action potential, thus inhibiting properties. It can be effective as a topical anesthetic,
nerve impulse propagation and leading to anesthesia. particularly in nasal and oral mucous membranes. However,
2. A patient with myasthenia gravis experiences muscle due to its potential for abuse and cardiovascular side effects,
weakness due to the autoimmune destruction of its use is now limited.
acetylcholine receptors at the neuromuscular junction. Which 6. Edrophonium is a short-acting anticholinesterase drug used in
of the following drugs is commonly used to improve muscle the diagnosis of a specific neuromuscular disorder. For which
strength in these patients by inhibiting the breakdown of of the following conditions is edrophonium commonly used
acetylcholine? (Annual 2024) as a diagnostic test? (Annual 2024)
a) Levodopa a) Lambert-Eaton syndrome
b) Atropine b) Amyotrophic lateral sclerosis
c) Pyridostigmine c) Myasthenia gravis
d) Diazepam d) Guillain-Barré syndrome
Answer: c) Pyridostigmine Answer: c) Myasthenia gravis
Explanation: Pyridostigmine is an acetylcholinesterase Explanation: The edrophonium test (Tensilon test) involves
inhibitor. By inhibiting the enzyme that breaks down the intravenous administration of edrophonium, a short-
acetylcholine, it increases the concentration and duration of acting acetylcholinesterase inhibitor. In patients with
action of acetylcholine at the neuromuscular junction, myasthenia gravis, this temporarily increases acetylcholine
thereby improving muscle strength in patients with levels at the neuromuscular junction, leading to a brief
myasthenia gravis. improvement in muscle strength, which helps in diagnosis.
3. A potent neurotoxin produced by a bacterium blocks the 7. Which drug is commonly used for long-term skeletal muscle
release of acetylcholine at the neuromuscular junction, relaxation during intensive care or prolonged surgeries?
leading to muscle paralysis. Which of the following toxins has (Supply 2024)
this mechanism of action? (Annual 2024) a) Succinylcholine
a) Tetrodotoxin b) Vecuronium
b) Curare c) Lidocaine
c) Botulinum toxin d) Halothane
d) Sarin Answer: b) Vecuronium
Answer: c) Botulinum toxin Explanation: Vecuronium is a non-depolarizing muscle
Explanation: Botulinum toxin, produced by the bacterium relaxant ideal for long-term use, unlike succinylcholine which
Clostridium botulinum, acts by inhibiting the release of is used for short duration.
acetylcholine from presynaptic nerve terminals at the 8. After surgery under general anesthesia, a patient reports
neuromuscular junction, preventing muscle contraction and severe muscle pain. Which drug is the likely cause? (Supply
leading to flaccid paralysis. 2024)
4. During surgical procedures requiring muscle relaxation, a) Rocuronium
certain neuromuscular blocking agents are used that prevent b) Succinylcholine
muscle contraction without causing initial muscle c) Midazolam
fasciculations. Which of the following drugs is a non- d) Atropine
depolarizing muscle relaxant? (Annual 2024) Answer: b) Succinylcholine
a) Succinylcholine Explanation: Succinylcholine causes muscle fasciculations
b) Rocuronium leading to postoperative muscle pain, especially in young,
c) Tubocurarine muscular patients.
d) Suxamethonium 9. Which factor most significantly influences the onset and
Answer: c) Tubocurarine duration of action of an anesthetic drug? (Supply 2024)
Explanation: Tubocurarine is a classic example of a non- a) pH
depolarizing neuromuscular blocking agent. These drugs b) Protein binding
compete with acetylcholine for binding to nicotinic receptors c) Lipid solubility
at the neuromuscular junction, preventing depolarization and d) Temperature

444
Medico Express Block 2 DISEASE PREVENTION AND IMPACT

DISEASE PREVENTION AND IMPACT


MS-CM-001 Community Medicine and Public Health (Back Pain) Learning Objectives
Explain causes of low back pain
Describe prevention of low back pain

BACK PAIN
Causes of Low Back Pain
Cause Description
Muscle or Ligament Overstretching or tearing of muscles or ligaments supporting the spine, often due to sudden
Strain movements or improper lifting techniques.
Herniated Disc Protrusion of the soft inner material of an intervertebral disc through the tough outer layer, leading
to irritation of nearby nerves.
Degenerative Disc Degeneration of intervertebral discs with age, resulting in reduced cushioning, stiffness, and
Disease decreased flexibility in the spine.
Spinal Stenosis Narrowing of spaces within the spine, causing pressure on the spinal cord and nerves, leading to
pain, numbness, or weakness in the lower back and legs.
Spondylolisthesis Forward displacement of a vertebra over the one below it, often caused by degenerative changes,
fractures, or congenital abnormalities.
Osteoarthritis Degenerative joint disease affecting the spine, leading to pain, stiffness, and swelling in the lower
back.
Traumatic Injury Fractures, dislocations, or other injuries to the vertebrae resulting from accidents or falls, causing
severe back pain.
Sciatica Pain along the path of the sciatic nerve, usually caused by compression or irritation of the nerve
roots in the lumbar spine.
Skeletal Irregularities Abnormal spinal curvature such as scoliosis or lordosis, which can lead to strain on muscles and
ligaments, resulting in back pain.
Other Medical Kidney stones, infections, tumors in the spine or nearby organs, and other medical conditions that
Conditions can cause low back pain, though less common.

Prevention of Low Back Pain


Prevention Strategy Description
Regular Exercise Engage in physical activities like walking, swimming, yoga, or Pilates to strengthen core and
back muscles, improve flexibility, stability, and posture.
Maintain a Healthy Weight Keep weight within a healthy range to reduce strain on the lower back muscles and
ligaments.
Practice Proper Posture Sit, stand, and lift with proper posture to maintain spinal alignment and reduce the risk of
strain and injury.
Use Ergonomic Furniture and Utilize ergonomic chairs, desks, and computer setups to support proper posture and reduce
Equipment strain while working or sitting.
Avoid Prolonged Sitting or Alternate between sitting, standing, and walking throughout the day to prevent stiffness and
Standing muscle fatigue.
Practice Safe Lifting Lift heavy objects using proper techniques, bending at the knees, and keeping the back
Techniques straight to minimize the risk of injury.
Stay Hydrated and Maintain Drink plenty of water to keep spinal discs hydrated and consume adequate calcium and
Bone Health vitamin D to support bone health.
Manage Stress Practice stress-reduction techniques such as deep breathing, meditation, or yoga to alleviate
muscle tension and promote relaxation.
Quit Smoking Quit smoking to improve circulation and promote spinal health by reducing the risk of disc
degeneration and low back pain.

SUMMARY
Causes of Low Back Pain:
• Muscle/ligament strain
• Herniated disc

447
Medico Express Block 2 DISEASE PREVENTION AND IMPACT
• Degenerative disc disease
• Spinal stenosis
• Spondylolisthesis
• Osteoarthritis
• Trauma
• Sciatica
• Spinal deformities
• Other conditions (kidney stones, infections, tumors)
Prevention:
• Exercise regularly
• Maintain healthy weight
• Use proper posture
• Use ergonomic furniture
• Avoid prolonged sitting/standing
• Lift safely
• Stay hydrated, support bone health
• Manage stress
• Quit smoking

448
Medico Express Block 2 DISEASE PREVENTION AND IMPACT

MS-CM-002 Community Medicine and Public Health (Work related Musculoskeletal disorders) Learning
Objectives
Describe work related musculoskeletal disorders addition with its burden / epidemiology
Identify risk factors of Musculoskeletal disorders MSD at workplace Describe prevention of exposure to
risk factors related to workplace

Work-related musculoskeletal disorders (MSDs)


These are a group of painful conditions affecting muscles, tendons, ligaments, nerves, and joints. These disorders are caused or
exacerbated by work-related activities and ergonomic factors, such as repetitive movements, awkward postures, forceful
exertions, and prolonged periods of sitting or standing.
Types of Work-Related MSDs:
1. Repetitive Strain Injuries (RSIs): These injuries occur due to repetitive movements or overuse of specific muscles or joints,
leading to pain, stiffness, and reduced range of motion. Examples: Carpal tunnel syndrome, tendonitis, and bursitis.
2. Back Pain: Work-related factors such as heavy lifting, improper lifting techniques, and prolonged sitting or standing can
contribute to low back pain and spinal disorders. Examples: Herniated discs, degenerative disc disease, and spinal
stenosis.
3. Neck and Shoulder Pain: Poor workstation ergonomics, repetitive overhead tasks, and prolonged computer use can lead
to neck and shoulder pain, as well as conditions. Examples: Cervical spondylosis and rotator cuff injuries.
4. Upper Extremity Disorders: These include conditions affecting the hands, wrists, elbows, and forearms. Examples: Tennis
elbow, De Quervain's tenosynovitis, and trigger finger, often caused by repetitive gripping, typing, or assembly line work.
5. Lower Extremity Disorders: Prolonged standing, heavy lifting, and repetitive movements can contribute to lower limb
MSDs. Examples: Knee osteoarthritis, plantar fasciitis, and Achilles tendinitis.
Burden and Epidemiology:
• Prevalence: In Pakistan, industries such as agriculture, construction, manufacturing, healthcare, and textiles have been
identified as sectors with high rates of work-related MSDs.
• Economic Costs: These disorders result in significant economic burdens due to healthcare costs, lost productivity,
absenteeism, and disability compensation.
• Impact on Quality of Life: Work-related MSDs can have a profound impact on an individual's quality of life, causing pain,
functional limitations, and psychological distress.
• Occupational Groups at Risk: Certain occupations are at higher risk for work-related MSDs, including healthcare workers,
construction workers, assembly line workers, office workers, and those in manual labor professions.
• Age and Gender: While MSDs can affect individuals of all ages and genders, older workers and women may be at higher
risk due to age-related changes in musculoskeletal health and differences in occupational tasks and exposures.
• Preventive Measures: Effective prevention strategies include ergonomic interventions, workplace modifications, job
rotation, employee education and training, and early intervention programs aimed at identifying and addressing
ergonomic risk factors before they cause injury.
Identify Risk Factors of Musculoskeletal Disorders MSD
1. Repetitive Movements:
− Assembly line work,
− Typing,
− Using vibrating tools,
can strain muscles and tendons, leading to MSDs like tendonitis or carpal tunnel syndrome.
2. Awkward Postures:
− Bending,
− Twisting,
− Reaching overhead,
can put excessive strain on the musculoskeletal system and increase the risk of MSDs.
3. Forceful Exertions:
− Lifting or
− Carrying heavy objects without proper equipment or techniques,
can lead to acute injuries or cumulative trauma disorders in the muscles and joints.
4. Static Loading:
− Where muscles are contracted or held in a fixed position without movement,
can lead to fatigue, discomfort, and eventually MSDs.
5. Vibration:
− Jackhammers
− Power tools,

449
Medico Express Block 2 DISEASE PREVENTION AND IMPACT
can contribute to Vibration white finger (VWF) → condition that can cause white fingers in people who use vibrating
tools or are exposed to cold
6. Poor Ergonomics:
Inadequate workstation design, including poorly adjusted chairs, desks, computer monitors, or tools, can result in
awkward postures, repetitive movements, and increased risk of MSDs.
7. Heavy Physical Work: Jobs that involve
− Heavy lifting,
− Pushing,
− Pulling,
− Carrying loads beyond a worker's capacity
can strain muscles, ligaments, and joints, leading to MSDs like low back pain or shoulder injuries.
8. Lack of Rest Breaks
9. Psychosocial Factors:
− Workplace stress,
− Job dissatisfaction,
− High workload,
− Lack of control over tasks, and
− Poor social support
can contribute to mental and physical stress, exacerbating the risk of MSDs.
10. Environmental Factors:
− Extreme temperatures,
− Poor lighting,
− Noise,
− Other environmental factors
can affect worker comfort, concentration, and performance, potentially increasing the risk of MSDs.
Prevention of Exposure to Risk Factors
1. Risk Assessment:
− Physical inspections,
− Employee feedback,
− Analysis of past incidents.
2. Training and Education:
− Proper use of equipment,
− Handling hazardous materials,
− Emergency procedures. Ensure that
− Employees understand the risks associated with their tasks
− How to mitigate them.
3. Personal Protective Equipment (PPE):
− Provide appropriate PPE such as helmets, gloves, goggles, or respirators to employees based on the specific
hazards they may encounter.
− Ensure that PPE is properly fitted, maintained, and used correctly.
4. Engineering Controls:
− Ventilation systems to control airborne contaminants,
− Machinery guards to prevent contact with moving parts,
− Ergonomic designs to reduce physical strain.
5. Administrative Controls:
− Rotating job tasks to reduce repetitive motion injuries,
− Implementing safety protocols for handling hazardous materials,
− Scheduling regular breaks to prevent fatigue.
6. Regular Maintenance and Inspections:
− Maintain equipment and facilities regularly to ensure safe working conditions.
− Conduct routine inspections to identify any potential hazards or maintenance issues.
7. Emergency Preparedness:
− Develop and communicate emergency response plans for various scenarios, including fires, chemical spills, or
medical emergencies.
− Conduct regular drills to practice response protocols.
8. Promote a Safety Culture:
− Open communication about safety concerns,
− Provide opportunities for employees to participate in safety initiatives,
− Recognize and reward safe behaviors.
450
Medico Express Block 2 DISEASE PREVENTION AND IMPACT
9. Compliance with Regulations:
− Stay up-to-date with relevant safety regulations
− Compliance with legal requirements.
− Conduct regular audits or
− Inspections to assess compliance and
− Addressing any non-compliance issues
SUMMARY
Work-Related Musculoskeletal Disorders (MSDs) are painful conditions caused by repetitive work, poor posture, heavy
lifting, and stress. Common types include back pain, repetitive strain injuries, and neck/shoulder pain. They are common in
many industries and cause economic and health burdens.
Key risk factors: repetitive motions, awkward postures, forceful exertions, vibration, poor ergonomics, and stress.
Prevention: proper training, ergonomic workstations, use of PPE, job rotation, breaks, and safety culture.

451
Medico Express BLOCK 2 PAST OSPE

OSPE – BLOCK-2
GROSS ANATOMY
UPPER LIMB
Bones and Ligaments (From past OSPE)

467
Medico Express BLOCK 2 PAST OSPE

468
Medico Express BLOCK 2 PAST OSPE

469
Medico Express BLOCK 2 PERLS

PERLS
PERLS-1-08 Responsibility towards self and the profession
Punctuality:
In my personal opinion punctuality is the habit of being consistently on time, meeting deadlines and fulfilling your commitments.
Case study:
(From past ospe) You keep submitting your biochemistry assignments late after submission date. The HOD has asked you to plan
your study time and be more punctual.
ANSWER
1. Plan and prioritize
2. Make a things to do list
3. Manage your time by setting achievable tasks in a given period of time
4. Celebrate milestones to make punctuality a habit.
It shows:
1) Professionalism: Being punctual demonstrates ones commitment to ones profession
2) Reliability: punctual individuals are often regarded as trustworthy
3) Respect for others: Arriving on time shows respect for colleagues, clients and the value of their time
Demonstrate punctuality:
To demonstrate punctuality, I personally consider the following actions:
1. Arrive on Time: Aim to arrive at appointments, meetings, or work a few minutes early to allow for unseen delays
2. Plan and prioritize: Organize schedule and allocate time wisely
3. Set realistic deadlines: I personally believe that one should not go beyond ones limits *with special reference to medical
professionals
1. Arrive at meetings on time
2. Adhere to scheduled appointments
3. Prioritize your patients health
Self-evaluation
I have always maintained my attendance record and I always complete my tasks and appointments on time
ATTENDANCE RECORD
Copy your attendance
PERLS-1-09 Diversity-Equity Inclusion
TS have a hypothetical case study
MALE/FEMALE OF 45YRS AGE, RECENTLY DEPORTED FROM AMERICA TO PAKISTAN, VEGAN, barahman, IDENTIFIES
HERSELF/HIMSELF AS A MEMBER OF MIORITY ETHNIC GROUP, SHE IS FACING ANXIETY/SOCIAL ISOLATION/POST TRAUMMATIC
STRESSDISORDER ETC
POINTS TO BE NOTED
Cultural barrier; superstitious, sense of dressing, talking Religious considerations; vegan Language barrier; English ‘Hindi Financial
barrier; deported things you should cultural competences active listening inclusive language empathy collaboration
PERLS-1-10 Learnning Styles
There are different types of learners:
1. Visual learners: Learn with the help of diagrams, charts and graphs
2. Auditory learners: Learn thru discussions and explanations
3. Kinesthetic learners: Tactile leraners
4. Logical learners
5. Social learners
Responsibilities of being a learner:
495
Medico Express BLOCK 2 PERLS
1. Active participation: -Asking questions -Seeking clarifications -Contributing to group work
2. Effective Time Management; -Plan and prioritize -Making Schedule -Completing assignments
3. Self-motivation: Stay motivated and surround yourself with people who support and motivate you
4. Critical thinking: -Question assumptions -Analyzing various aspects of given information
5. Reflective practice: -Identify strengths, weaknesses and areas of improvement
6. Adaptibility
7. Ethical conduct
Responsibilities to oneself:
1. Self awareness
2. Self care
3. Setting boundaries
4. Goal setting
5. Accountability

PERLS1-11
Responsibilities Of a doctor Code of conduct:
It outlines the expected behavior and principles that individuals within an organization, community or a group are expected to
adhere.
Responsibilities of a doctor:
1. Patient care: -Provide complete information to the patient about his/her disease.
USE YOUR BRAINS:
B – BENEFITS
R – RISKS
A – ADVANTAGES
I – INTITUION
N – Nothing (respect the patient's right to choose)
2. Professionalism: -Integrity -Honesty -Adhere to ethical guidelines
3. Ethical practice:
4. Accountability
5. Communication: Effective communication is the best skill a doctor can have. Dont use medical jargon

PERLS 1-12 Digital Ethics


LinkedIn profile
Develop a comprehensive LinkedIn profile highlighting your education, work experience and skills. Connect with professional in
your field and participate in relevant conversations
Case study (From past ospe)
Some pictures of your friend have been leaked in a group how would you manage this?
Answer:
1. Get the pictures removed from group by contacting the admin
2. Request others to take this matter seriously
3. Contact cyber security department
4. Register a complaint
5. Awarness about cyber crimes
Ethical challenges and solutions
1. privacy concerns, set clear boundaries for what information you share online REVIEW PRIVACY SETTINGS
2. authenticity, be transparent about your achievements, avoid exaggerations, maintain honesty in digital interactions
3. impersonation and theft, secure passwords, BEWARE of suspicious activities
4. maintaining professionalism, avoid controversial discussions and represent yourself in a manner consistent with your
professional aspirations

496

You might also like